You are on page 1of 84

“c02VectorsInThreeDimensions_print” — 2019/8/7 — 6:51 — page 22 — #1

2 Vectors in three dimensions


2.1 Overview
Until this point, the study of vectors has been
restrained to only two dimensions. By
extending our knowledge to three dimensions,
the list of applications is endless. The product
of two vectors can extend from simply being a
scalar quantity (dot product), to that of a vector
product which is a vector. The vector product is
valuable in determining areas and volumes of
specific shapes, the turning force (torque)
applied to an object at a specific point and is
central in developing the equations of planes.
By extending our study of vectors to three
dimensions, it allows us to strengthen our
understanding of Cartesian equations and expand
our knowledge through the introduction to
parametric equations.
Parametric equations link our conventional understanding of equations of two variable with a third,
known as a parameter. The famous Brachistochrone problem incorporates three variables and was posed by
the Mathematician James Bernoulli (1654–1705). Bernoulli was determined to identify the path (x, y) in
which a particle could slide from two points (P and Q) in the least possible time (t). Challenging the likes of
fellow scholars such as Newton and L’Hopital, Bernoulli found that if the particle followed a path in the
shape of an inverted cycloid, it would reach its destination in the shortest possible time. Applications of
equations involving the parameter t are inherent in modelling problems such as the vector equation of a
straight line in three dimensions and determining points of collision between the paths of two or more
particles.

LEARNING SEQUENCE
2.1 Overview
2.2 Introduction to vectors in three dimensions
2.3 Geometric proofs using vectors
2.4 Cartesian and parametric equations
2.5 The vector equation of a straight line
2.6 The vector product
2.7 Applications of vectors
2.8 Review: exam practice
Fully worked solutions for this chapter are available in the Resources section of your eBookPLUS at
www.jacplus.com.au.

22 Jacaranda Maths Quest 12 Specialist Mathematics Units 3 & 4 for Queensland


“c02VectorsInThreeDimensions_print” — 2019/8/7 — 6:51 — page 23 — #2

2.2 Introduction to vectors in three dimensions


Introduced in Unit 1, vectors have thus far only been considered in two-dimensional form. In this chapter,
following a review of two-dimensional vectors, our study of vectors will be extended to three-dimensions.

2.2.1 Review of vectors in two-dimensions


Scalar and vector quantities
A scalar quantity is a real number only, it has magnitude, but no direction. Scalar quantities include
temperature, mass and time.
A vector is a quantity that has magnitude and direction. Vectors include weight in a particular direction
and displacement with a specific direction.
Vectors are directed line segments with a tail (start) and head (end). In the figure below, the head of the
vector is at point B (indicated with an arrow), while the tail is at point A. In this instance, the vector could be
expressed in a variety of forms.

Various forms of vector notation



AB = ~u = u

B
u
~

The tilde symbol (~) is commonly used because drawing bold letters is often difficult.

Equality of vectors
Two vectors are equal if both their magnitude and direction are equal. w
~
In this figure, the following statements can be made:
~ = ~v (direction and magnitude equal)
u u
~
~u ≠ w~ (directions are not equal) v
u ≠
~ ~ (magnitudes are not equal)
z ~
z
~

Addition and subtraction of vectors B


The triangle rule of vectors can be applied in both the addition and
subtraction of vectors. u
~
v
~
To add two vectors, take the tail of one vector and join it to the head
of another. The result of this addition is the vector from the tail of the first
vector to the head of the second vector. A w=u+v C
If ~u is the vector from A to B, then −~u is the vector from B to A. ~ ~ ~

To subtract two vectors, add the negative of the second vector –u


A ~
to the first vector. For example, A − B = A + (−B).
B
u
~

CHAPTER 2 Vectors in three dimensions 23


“c02VectorsInThreeDimensions_print” — 2019/8/7 — 6:51 — page 24 — #3

Multiplying a vector by a scalar


Multiplying a vector by a positive number (scalar) affects only the magnitude N
of the vector, not the direction.
W E
If the scalar is negative, then the direction is reversed.
S
Cartesian form of a vector in two-dimensions
Thus far, points have been considered in 2-dimensions. A point, P (x, y), is u 3u –2u
~ ~ ~
determined by its distances from two mutually perpendicular axes.
On the Cartesian plane, a position vector is the directed line segment joining
the origin with the point P. It defines a point, P, by magnitude and direction
relative to the origin, O, and can be written as ⃗OP or ~u. y
The position vector ~u is the sum of the two directional unit vectors, î and P (x, y)
ĵ where: u
~
• î is a unit vector along the x-axis, with magnitude 1. ĵ
• ĵ is a unit vector along the y-axis, with magnitude 1.
For example, if the point C has coordinates (5, 4), then ~u = 5î + 4ĵ. O î x

y
C (5, 4)

O 5î x

This is the Cartesian form of a vector, and is given by,


~u = OP = xî + yĵ
The position vector, ~u = 5î + 4ĵ can also be expressed as an ordered pair (5, 4) or in column vector
5
notation as, [ ] .
4

The magnitude of a vector


By using Pythagoras’ theorem on a position vector, we can determine its length, or magnitude.
The magnitude of the vector ~u = xî + yĵ is also called the norm and is denoted as |~u| or r.

|~u| = x2 + y2

The magnitude of ~u = 5î + 4ĵ can be calculated using the above formula



|~u| = 52 + 42

= 41

24 Jacaranda Maths Quest 12 Specialist Mathematics Units 3 & 4 for Queensland


“c02VectorsInThreeDimensions_print” — 2019/8/7 — 6:51 — page 25 — #4

The unit vector


The magnitude of a vector is also used to determine the unit vector. If ~u is the vector, then the unit vector,
u,̂ is given by,
denoted by ~

û = ~u
~ |u|
~

For example, the unit vector for ~u = 5î + 4ĵ is:

û = ~u
~ |u|
~
5î + 4ĵ
= √
41
5 4 1
= √ î+ √ ĵ or √ (5î + 4ĵ)
41 41 41

Polar form
In polar form, the vector ~u = xî + yĵ, can be expressed in terms of its magnitude |~u| and direction 𝜃. The
[ ]
polar form of a vector ~u is r, 𝜃 , where:
• r is the magnitude of ~u
• 𝜃 is the anti-clockwise
( )
angle formed from the positive x-axis (polar axis)
y
• 𝜃 = tan−1
x
The position vector ~u = 5î + 4ĵ can be expressed in polar form as shown.
y
~u = 5î√+ 4ĵ C (5, 4)
|~u| = 41 (previously calculated)
( )
4 │~u│
−1
𝜃 = tan
5
𝜃 ≈ 0.6747 radians
θ
𝜃 ≈ 38.66°
[√ ] O x
∴ ~u = 51, 38.66°

Resolving vectors into components


If the two-dimensional vector ~u makes an angle of 𝜃 with the positive x-axis and it has a magnitude of r,
then we can find its x- and y-components using the formulas:

x = |r| cos 𝜃
y = |r| sin 𝜃

For example, the vector in polar form ~v = [5, 233.13°], can be converted to component (Cartesian) form:

~v = 5 cos(233.13°) î + 5 sin(233.13°) ĵ
= −3î − 4ĵ

CHAPTER 2 Vectors in three dimensions 25


“c02VectorsInThreeDimensions_print” — 2019/8/7 — 6:51 — page 26 — #5

2.2.2 Introduction to vectors in three-dimensions


z
Cartesian coordinates for three-dimensional planes
In the three-dimensional coordinate system, the z-axis
exists perpendicular to both the x-and y-axes at the origin.
The three-dimensions are shown with the x-axis coming
out of the page and the y- and z-axes lying flat on the page.

The space consists of 8 regions called octants that z


are divided by three perpendicular planes. Points in xz Plane
the first octant have positive values for x, y and z.

xy Plane
y

yz Plane

The orientation of the axes in the diagram indicates that it z


is a right-handed coordinate system. Systems can be either
right- or left-handed in orientation, however the right-handed
is the preferred option.
As the fingers on your right-hand curl from the x-axis
to the y-axis, your thumb points in the direction of the
positive z-axis.

26 Jacaranda Maths Quest 12 Specialist Mathematics Units 3 & 4 for Queensland


“c02VectorsInThreeDimensions_print” — 2019/8/7 — 6:51 — page 27 — #6

The following arrangements are also examples of right-handed systems.

x y

z
y z x

In three-dimensional space the coordinates (x, y, z) define a point P (x, y, z), with each point made up of
three components. Points A (4, −2, 0), B (1, 5, 0), C (0, 0, 2) and D (−1, −4, 2) are shown in the diagram.
z

D(–1, –4, 2)
4
–6

–6 –4
2 C(0, 0, 2)
–4
–2
–2

0
2
2
4
A(4, –2, 0)
4 –2 6
B(1, 5, 0) 8
6
y
8 –4

–6

CHAPTER 2 Vectors in three dimensions 27


“c02VectorsInThreeDimensions_print” — 2019/8/7 — 6:51 — page 28 — #7

The location of a point relative to the origin can be z

expressed as the position vector ⃗ OP , in terms of the


unit vectors, î, ĵ and k̂. These are known as the
standard unit vectors:
Let î be a unit vector in the positive direction, parallel to C (0, 0, 1)
the x-axis, with magnitude 1.

Let ĵ be a unit vector in the positive direction, parallel ĵ B (0, 1, 0)
to the y-axis, with magnitude 1. O
y
Let k̂ be a unit vector in the positive direction, parallel to î
the z-axis, with magnitude 1.
A (1, 0, 0)
Points A, B and C, can be expressed in component
form in terms of the standard unit vectors î, ĵ and k̂. x

î = (1, 0, 0) ∴⃗
OA = î
ĵ = (0, 1, 0) ∴⃗
OB = ĵ
k̂ = (0, 0, 1) ⃗
∴O C = k̂

In three-dimensions, a position vector beginning at the origin O (0, 0, 0) and terminating at the point
P (x, y, z) is defined as,


OP = xî + yĵ + zk̂
~u = xî + yĵ + zk̂

A vector expressed in terms of î, ĵ and k̂ is said to be in component form (or Cartesian form), and the
process is called the resolution of vectors.
The position vector ⃗ OP can be determined using a vector sum. z

OP = ⃗
∴ ⃗ OA + ⃗
AB + ⃗
BP
= x1 î + y1 ĵ + z1 k̂
z1
P (x1, y1 ,z 1)


Magnitude and unit vectors in three-dimensions. y1
O
The magnitude of the three-dimensional vector î ĵ y

~u = xî + yĵ + zk̂ is defined by the formula,


x1

√ A
|~u| = x2 + y2 + z2 B

x
If ~u is the vector, then the unit vector denoted by ~u,̂ is
given by,

û = ~u
~ |u|
~

28 Jacaranda Maths Quest 12 Specialist Mathematics Units 3 & 4 for Queensland


“c02VectorsInThreeDimensions_print” — 2019/8/7 — 6:51 — page 29 — #8

WORKED EXAMPLE 1
If the point P has the coordinates (−7, 3, −4), determine:
a. The vector ⃗OP
b. The magnitude of ⃗ OP
c. The unit vector parallel to ⃗
OP
THINK WRITE
a. The î, ĵ, k̂ components of the vector ⃗OP correspond a. P (−7, 3, −4) has coordinates
to the x-, y- and z-coordinates respectively. x = −7, y = 3, z = −4

OP = −7î + 3ĵ − 4k̂

b. Recall that the magnitude of the vector b. | ⃗
OP | = (−7)2 + (3)2 + (−4)2
√ √
~u = xî + yĵ + zk̂ is given by |~u| = x + y + z .
2 2 2
= 74

p
c. 1. Let ⃗
OP = p, the unit vector parallel to ⃗
OP is c. p̂ = ~
~ ~ |p|
p ~
notated as p.̂ Recall the unit vector formula p̂ = ~ . −7î + 3ĵ − 4k̂
~ ~ |p| = √
~ 74
1 ( )
2. The unit vector is often expressed as a scalar =√ −7î + 3ĵ − 4k̂
multiple of p as shown here. 74
~

TI | THINK WRITE CASIO | THINK WRITE


b. 1. On a Calculator b.1. On a Run-Matrix screen,
page, select MENU 7: select MAT/VCT
Matrix Vector 7: Norms
1: Norm

2. Complete the entry line 2. To switch between the


([−7, 3, −4]) vector and matrix
screens, press the
M ⇔ V button.
Press the EXE button to
define the Vct A.

3. Press the ENTER 3. Complete the entry


button. line as
The answer appears on 1
the screen. 3
Press the EXE button.

CHAPTER 2 Vectors in three dimensions 29


“c02VectorsInThreeDimensions_print” — 2019/8/7 — 6:51 — page 30 — #9

4. Complete the entry


line as
–7 3 –4
Press the EXIT button.

5. Press the EXIT button


to return to the
Run-Matrix screen.
Select OPTN MAT/VCT
⊳ (x3 times)
The screen should appear
as shown.

6. Select, Norm (⊳
(x3 times) Vct ALPHA
X, 𝜃, T)

7. Press the EXE button to


complete the calculation.
The answer appears on
the screen.

Note that the TI calculator can only display the magnitude in decimal form.

The vector between two points in three-dimensions z

In the diagram opposite, ~a = ⃗ OA is the position vector of point A


A (x1, y1, z1)
and ~b = ⃗OB is the position vector of point B. The vector
a
describing the location of A relative to B denoted by ⃗BA , is ~ ⃪
AB
found using vector addition as −~b + ~a or ~a − ~b.
The vector describing the location of B relative to y
O
A denoted by ⃗ AB , shown, is ~b − ~a.
b
If A and B are points defined by the position vectors ~
B (x2, y2, z2)
~a and ~b respectively, then x


AB = ⃗AO + ⃗ OB

= − OA + ⃗ OB z
A (x1, y1, z1)

= OB − OA ⃗
b
= ~b − ~a ~
a
~
Addition and subtraction of three-dimensional vectors
in component form a+b
~ ~ y
Given, ~a = (x1 , y1 , z1 ) and ~b = (x2 , y2 , z2 ) O
Addition of vectors in component form is: b
( ) ( ) ~
a +
~ ~ b = x 1 î + y 1 ĵ + z1 k̂ + x2 î + y 2 ĵ + z2 k̂ B (x2, y2, z2)
x
= (x1 + x2 ) î + (y1 + y2 ) ĵ + (z1 + z2 ) k̂

30 Jacaranda Maths Quest 12 Specialist Mathematics Units 3 & 4 for Queensland


“c02VectorsInThreeDimensions_print” — 2019/8/7 — 6:51 — page 31 — #10

Subtraction of vectors in component form is: z


( ) ( )
~a − ~b = x1 î + y1 ĵ + z1 k̂ − x2 î + y2 ĵ + z2 k̂
–~b
A (x1, y1, z1)
= (x1 − x2 ) î + (y1 − y2 ) ĵ + (z1 − z2 ) k̂
a–b a
~ ~ ~

y
O
b
~
B (x2, y2, z2)
x

WORKED EXAMPLE 2
Three points, A, B and C have the coordinates (−3, 7, −1), (2, −2, 11) and (8, −7, 12) respectively.
Calculate:
a. The vector ⃗
AB

b. The vector BC
⃗ − 4⃗
c. The vector sum u = 3 BC AB
~

THINK WRITE
a. 1. Define the vector ⃗ OA . a. ⃗
OA = −3î + 7ĵ − k̂

2. Define the vector OB . ⃗
OB = 2î − 2ĵ + 11k̂
⃗ ⃗
3. AB is determined by subtracting OA from OB . ⃗ ⃗ OB − ⃗
AB = (⃗ OA ) ( )
= 2î − 2ĵ + 11k̂ − −3î + 7ĵ − k̂
= (2 + 3)î + (−2 − 7)ĵ + (11 + 1)k̂
= 5î − 9ĵ + 12k̂
b. Define the vector O ⃗ C. ⃗
b. O C = 8î − 7ĵ + 12k̂

BC is determined by subtracting ⃗ OB from O⃗ C. ⃗
BC = (O⃗ C−⃗ OB ) ( )
= 8î − 7ĵ + 12k̂ − 2î − 2ĵ + 11k̂
= (8 − 2)î + (−7 + 2)ĵ + (12 − 11)k̂
= 6î − 5ĵ + k̂
⃗ ⃗
c. 1. The vectors BC and AB have been defined. c. u = 3 ⃗BC − 4 ⃗AB
~ ( ) ( )
Substitute these into the vector sum = 3 6î − 5ĵ + k̂ − 4 5î − 9ĵ + 12k̂
⃗ ⃗
~u = 3 BC − 4 AB .
2. Simplify the vector sum by multiplying the = 18î − 15ĵ + 3k̂ − 20î + 36ĵ − 48k̂
respective vectors by the scalar values. Define ~u = −2î + 21ĵ − 45k̂
by applying the rules for adding and subtracting
vectors as components.

Scalar multiples and parallel vectors


When a vector is multiplied by a scalar, 𝜆, each coefficient is multiplied by the scalar.
( )
𝜆~a = 𝜆 x1 î + y1 ĵ + z1 k̂
= 𝜆x1 î + 𝜆y1 ĵ + 𝜆z1 k̂

CHAPTER 2 Vectors in three dimensions 31


“c02VectorsInThreeDimensions_print” — 2019/8/7 — 6:51 — page 32 — #11

Two vectors can be defined as parallel if one is a scalar multiple of the other. That is, ~b is parallel to ~a if
~ 𝜆~a.
b =

Equality of vectors.
Consider two vectors, ~a = x1 î + y1 ĵ + z1 k̂ and ~b = x2 î + y2 ĵ + z2 k̂. The vectors are equal if and only if,
x1 = x2 , y1 = y2 and z1 = z2 .

The midpoint of a vector in three-dimensions


Consider the two vectors, z
⃗ ⃗
~a = OA and ~b = OB A (x1, y1, z1)
The midpoint, M, of the line segment AB, can be
represented by the vector,
a

OA + ⃗ OB
~

OM = . Alternatively, it can be defined as, M y
2
O
1
⃗OM = ⃗ OA + ⃗ AB . b
2 ~
B (x2, y2, z2)
x

WORKED EXAMPLE 3
The location of two points are defined by P (4, 8, −2) and Q (12, −3, 6).
Calculate the position vector ⃗
OM , where M is the midpoint of the line segment PQ.

THINK WRITE
1. Define the vector ⃗
OP . ⃗
OP = 4î + 8ĵ − 2k̂

2. Define the vector O Q. ⃗
O Q = 12î − 3ĵ + 6k̂

OP + O ⃗ Q
3. Recall the rule that defines a midpoint as a ⃗OM =
vector. 2
( ) ( )
4î + 8ĵ − 2k̂ + 12î − 3ĵ + 6k̂
4. Calculate ⃗OM by substituting ⃗ ⃗
OP and O Q =
into the formula. 2
16î + 5ĵ + 4k̂
=
2
5
= 8î + ĵ + 2k̂
2
5. Conclude with a statement. The midpoint between the points P and Q
is represented by the position vector
⃗ 5
OM = 8î + ĵ + 2k̂.
2

2.2.3 The dot product


Review of the dot product
When a vector is added or subtracted to another vector, or multiplied by a scalar quantity, the result is a
vector. However, when vectors are multiplied this can either be in the form of the dot (scalar) product
or vector (cross) product. The dot product results in a scalar quantity, whereas the vector product results in
a vector.

32 Jacaranda Maths Quest 12 Specialist Mathematics Units 3 & 4 for Queensland


“c02VectorsInThreeDimensions_print” — 2019/8/7 — 6:51 — page 33 — #12

The dot or scalar product is also known as the inner product and can be
calculated using two different methods. The first method is used if the magnitude of
a
two vectors and the angle, 𝜃, between them are known. ~

θ
~a · ~b = |~a||~b| cos 𝜃 b
~

Where 𝜃 is the angle between the tail of the vectors ~a and ~b.

WORKED EXAMPLE 4
Examine the diagram and determine ~a · ~b.

a
~ b
~
θ = 145°
15 12

THINK WRITE
1. The magnitude of the two vectors and the angle |~a| = 15, |~b| = 12, 𝜃 = 145°
between is given.
2. Recall the dot product formula and substitute in
~a · ~b = |~a||~b| cos 𝜃
relevant values. = 15 × 12 × cos(145°)
= −147.45

The dot (scalar) product of vectors expressed in component form


The second method of calculating the dot (scalar) product is used with the vectors in component form.
Consider the standard unit vectors î, ĵ and k̂. The magnitude of each vector is 1 and can be expressed as
|~î| = 1, |ĵ| = 1 and |k̂| = 1. Begin by examining î · î in detail.
~
• The angle between î and î is 0°.
• cos(0°) = 1
By definition î · î is
î · î = |î||î| cos (0°)
= |î|2 (where |î| = 1)
=1
Extrapolating from î to ĵ and k̂:
î · î = 1, ĵ · ĵ = 1 and k̂ · k̂ = 1
The unit vectors are mutually perpendicular to each other, that is the angle between î, ĵ and k̂ are all 90°.
Given that cos(90°) = 0 and the dot product is commutative, it follows that î · ĵ = ĵ · î = 0, ĵ · k̂ = k̂ · ĵ = 0
and î · k̂ = k̂ · î = 0.
This allows us to calculate the dot product for any two vectors, ~a = x1 î + y1 ĵ + z1 k̂ and
~b = x2 î + y2 ĵ + z2 k̂, where x, y and z are(
constants.
) ( )
~a · ~b = x1 î + y1 ĵ + z1 k̂ · x2 î + y2 ĵ (+ z2 k̂)
= x1 x2 (î · î) + y1 y2 ( ĵ · ĵ) + z1 z2 k̂ · k̂
= x1 x2 + y1 y2 + z1 z2

We only need to multiply the corresponding x-, y- and z- components of two vectors to find their
dot product.

CHAPTER 2 Vectors in three dimensions 33


“c02VectorsInThreeDimensions_print” — 2019/8/7 — 6:51 — page 34 — #13

WORKED EXAMPLE 5

Given the vectors, a = 5î + 2ĵ − 8k̂ and b = −2î + ĵ − 3k̂, determine ~a · ~b.


~ ~
THINK WRITE
( ) ( )
1. Substitute for the given vectors. a
~ ~· b = 5î + 2ĵ − 8k̂ · −2î + ĵ − 3k̂
2. Add the products of the corresponding x−, y−, z−
~a · ~b = (5 × −2) + (2 × 1) + (−8 × −3)
components. = −10 + 2 + 24
= 16
3. State the final result. ~ ~ = 16
a · b

TI | THINK WRITE CASIO | THINK WRITE


1. On a Calculator page, 1. On a Run-Matrix
select MENU screen, select
7: Matrix Vector MAT/VCT
C: Vector Define Vectors A and B
3: Dot Product as shown in Worked
Press the ENTER example 1b.
button.

2. Complete
([ ]the
[ entry line]) 2. On a Run-Matrix screen,
5, 2, −8 , −2, 1, −3 select OPTN MAT/VCT
⊳ (x3 times) DotP(

3. Press the ENTER 3. Complete the entry


button. line as
The answer appears on Vct ALPHA X, 𝜃, T,
the screen. Vct ALPHA Log )

4. Press the EXE button to


perform the calculation.
The answer appears on
the screen.

Determining the angle between two vectors


Given two formulas for calculating the dot product:

~a · ~b = |~a||~b| cos 𝜃
~a · ~b = x1 x2 + y1 y2 + z1 z2

they can be combined to give:

x1 x2 + y1 y2 + z1 z2 = |~a||~b| cos 𝜃

34 Jacaranda Maths Quest 12 Specialist Mathematics Units 3 & 4 for Queensland


“c02VectorsInThreeDimensions_print” — 2019/8/7 — 6:51 — page 35 — #14

Rearranging this formula gives the angle between two vectors.

The angle between two vectors


x x + y1 y2 + z1 z2 a·b
cos 𝜃 = 1 2 or cos 𝜃 = ~ ~
|~a||~b| |~a||~b|

The dot product of perpendicular vectors


It was shown that the dot product of perpendicular unit vectors is equal to zero.
î · ĵ = ĵ · î = 0, ĵ · k̂ = k̂ · ĵ = 0 and î · k̂ = k̂ · î = 0
This is true for any two, non-zero vectors, ~a and ~b. If the two vectors ~a and ~b are perpendicular
(orthogonal) the angle between them is 90°, and the equation ~a · ~b = |~a||~b| cos 𝜃 becomes,

~a · ~b = |~a||~b| cos(90°)
= |~a||~b| × 0 (Recall cos(90°) = 0)
=0
If ~a · ~b = 0, then the vectors ~a and ~b are perpendicular.

Properties of the dot product


• The dot product is a scalar. It is the result of multiplying scalar quantities.
• If the magnitude of two vectors and the angle, 𝜃, between them are known it is calculated
by: ~a · ~b = |~a||~b| cos 𝜃. If the vectors are in component form, it is calculated by:
~a · ~b = x1 x2 + y1 y2 + z1 z2
• The order of multiplication is unimportant ( )(commutative property): ~a · ~b = ~b · ~a
• The dot product is distributive: ~ ~ ~ = ~c · ~a + ~c · ~b
c · a + b
• Since the angle between the same vector and itself is 0°: ~a · ~a = |~a|2
• If ~a · ~b = 0, then the vectors ~a and ~b are perpendicular.

WORKED EXAMPLE 6

The vectors given by, a = 6î + yĵ − k̂ and b = −3î + 4ĵ + 2k̂, are perpendicular.


~ ~
Calculate the value of y.
THINK WRITE
( ) ( )
1. Write ~a · ~b as the dot product of the two vectors ~a · ~b = 6î + yĵ − k̂ · −3î + 4ĵ + 2k̂
in component form.
2. Add the products of the corresponding x-, y-, z-
~a · ~b = (6 × −3) + (y × 4) + (−1 × 2)
components. = −18 + 4y − 2
= 4y − 20 [1]
3. State the dot product relationship of ~a is perpendicular to ~b
perpendicular vectors. ∴ ~a · ~b = 0 [2]
4. Substitute the initial solution for a · b [1] into
~ ~ ~a · ~b = 0
[2] to solve for y. 4y − 20 = 0
20
y=
4
5. The solution is stated. ∴y=5

CHAPTER 2 Vectors in three dimensions 35


“c02VectorsInThreeDimensions_print” — 2019/8/7 — 6:51 — page 36 — #15

2.2.4 Review of vector projections (vector resolutes)


We have resolved vectors parallel and perpendicular to the x- and
y-axes. Now we will examine the more generalised process of
resolving one vector parallel and perpendicular to another vector.
b
Consider the two vectors ~a and ~b, shown. The angle between ~
them, as for a dot product, is given by 𝜃. It can be shown that ~b is
made up of a projection acting in the direction of ~a and another
projection acting perpendicular to ~a. 𝜃
a
~
The first step is to find the projection in the direction of ~a.

The scalar projection (scalar resolute)


• The length OA is the scalar resolute of ~b on ~a.
• It effectively indicates ‘how much’ of ~b is in the direction of ~a.
Using trigonometry OA = |~b| cos 𝜃, and the dot product
~a · ~b = |~a||~b| cos 𝜃:
b
~
~a · ~b = |~a||~b| cos 𝜃
~a · ~b = |~a|OA 𝜃
a·b ~a O a A
∴ OA = ~ ~ , where ~â =
~
|~a| |~a|
= ~â · ~b

a·b
The scalar resolute of ~b on ~a is given by ~â · ~b or ~ ~ , where ~â is the unit vector in the direction of ~a.
|~a|

The scalar resolute of ~b on ~a is not equal to the scalar resolute of ~a on ~b.

Vector projection (vector resolutes) b


Consider now the vector joining O to A. Its magnitude is just the scalar ~ b
~⊥
resolute ~â · ~b, while its direction is the same as ~a, that is ~a. This quantity
is called the vector resolute of ~b parallel to ~a and is denoted by the θ
symbol ~b∥ . O b A a
~‖ ~

The vector resolute of ~


b parallel to ~a is given by
( )
~b∥ = ~â · ~b ~â

Consider the geometry of the above figure. The original vector ~b is the sum of ~b⊥ and ~b ∥ . The vector
resolute of ~b perpendicular to ~a can be calculated simply as shown below.
~b = ~b ∥ + ~b ⊥ (by addition of vectors)
~b ⊥ = ~b − ~b ∥ (by rearranging the vector equation)

The vector resolute of ~


b perpendicular to ~a is given by
( )
~b ⊥ = ~b − ~â · ~b ~â

36 Jacaranda Maths Quest 12 Specialist Mathematics Units 3 & 4 for Queensland


“c02VectorsInThreeDimensions_print” — 2019/8/7 — 6:51 — page 37 — #16

WORKED EXAMPLE 7

Let ~a = 8î − 6ĵ + 5k̂ and ~b = −6î + 2ĵ − k̂. Determine:


a. the scalar resolute of b on a
~ ~
b. the vector resolute of b parallel to a, namely b∥
~ ~ ~
c. the vector resolute of b perpendicular to a, namely b ⊥
~ ~ ~

THINK WRITE

a. 1. Calculate the magnitude of ~a. a. |a| = (8)2 + (−6)2 + (5)2

= 125

=5 5

2. Calculate the unit vector in the direction a ̂ = ~a


~ |a|
of a, ~a,̂ by dividing ~a by |~a|. (~ )
8î − 6ĵ + 5k̂
= √
( 5
5 )
8î 6ĵ 5k̂ ( )
3. Determine the scalar resolute of ~b on ~a, ~â · ~b = √ − √ + √ · −6î + 2ĵ − k̂
by substituting into the formula ~â · ~b. 5 5 5 5 5 5
−48 −12 −5
4. Simplify ~â · ~b using dot product laws for ~â · ~b = √ + √ + √
unit vectors. 5 5 5 5 5 5
65
=− √
5 5
13
= −√
5
( )
Determine b. b ∥ = â · b â
b. 1.
( )b∥ by recalling the formula
~ ~ ~ ~ ~
~b∥ = ~â · ~b ~â ( )( )
13 8î 6ĵ k̂
= −√ √ − √ +√
5 5 5 5 5 5

104 78 13
2. Simplify. =− î+ ĵ − k̂
25 25 5
( )
c. 1. Determine ~b ⊥ by recalling the formula c. b ⊥ = b − â · b â
~ ~ ~ ~ ~
~ ⊥ = ~b − ~b∥
b
( )
( ) 104 78 13
2. Substitute in the vectors, b ⊥ and b ∥ .
~ ~ ~b ⊥ = −6î + 2ĵ − k̂ − − 25 î + 25 ĵ − 5 k̂
46 28 8
3. Simplify. ~b ⊥ = − 25 î − 25 ĵ + 5 k̂

2.2.5 Three dimensional vectors in polar form


[ ]
In two-dimensions, a vector in polar form was defined by ~u = r, 𝜃 , where r is the magnitude of ~u and 𝜃 is
the anti-clockwise angle formed from the positive x-axis.
In three dimensions, the point P (x, y, z) can be defined by the position vector, ⃗
OP = ~u.

CHAPTER 2 Vectors in three dimensions 37


“c02VectorsInThreeDimensions_print” — 2019/8/7 — 6:51 — page 38 — #17

z
The vector, ~u, can then be represented in polar
[ ]
form ~u = r, 𝜃, 𝜑 where, P (x, y, z)
• r is the magnitude of ~u, where
√ u = [r, θ, φ]
~
r ≥ 0 and is given by r = x2 + y2 + z2 .
• 𝜃 (azimuth angle) is the anti-clockwise r
angle formed from the positive x-axis to
z
the projection of ~u onto the xy-plane,
where 0° ≤ 𝜃 ≤ 360°. O φ
y
• 𝜑 (altitude angle) is the angle between the line x
θ
a
segment ⃗ OP and the xy-plane, where
−90° ≤ 𝜑 ≤ 90°. y
The[ relationship
] between Cartesian (x, y, z) and
polar r, 𝜃, 𝜑 coordinates can be examined by x
referring to the diagram to the right. The findings will
be useful to convert between one form to the other.
Two trigonometric ratios can be formed by considering the triangle defining the altitude angle, 𝜑.

z a
sin 𝜑 = cos 𝜑 =
r r
r
∴ z = r sin 𝜑 [1] ∴ a = r cos 𝜑 [2]
z

φ
a

The second triangle forms a further three trigonometric ratios in terms of 𝜃, a, x and y:

y x y
sin 𝜃 = cos 𝜃 = tan 𝜃 = [5] θ
a a x
a
∴ y = a sin 𝜃 [3] ∴ x = a cos 𝜃 [4] x

x and y can be expressed in terms r and the two angles, through the process of substitution.
Substituting [2] into [4] and substituting [2] into [3] gives
x = r cos 𝜑 cos 𝜃 y = r cos 𝜑 sin 𝜃
These formulae are useful when working with vectors in three-dimensions. A summary of the necessary
rules to remember are detailed below.

[ ] ( )
u = r, 𝜃, 𝜑 = xî + yĵ + zk̂
When considering the three-dimensional vector, ~
z
x = r cos 𝜑 cos 𝜃 sin 𝜑 =
√ r
r = x2 + y2 + z2 y = r cos 𝜑 sin 𝜃
y
z = r sin 𝜑 tan 𝜃 =
x

38 Jacaranda Maths Quest 12 Specialist Mathematics Units 3 & 4 for Queensland


“c02VectorsInThreeDimensions_print” — 2019/8/7 — 6:51 — page 39 — #18

[ ]
Vectors expressed in the polar form, ~u = r, 𝜃, 𝜑 , are often related to the spherical coordinate system
which may be used to identify points on the Earth’s surface.

WORKED EXAMPLE 8

a. u = [10, 30∘, 45∘] in component form, (xî + yĵ + zk).


Express ~ ̂ Give your answer as an exact value.
b. Determine the altitude angle, to three decimal places, between the vector, ~v = −12î + 9ĵ − 8k̂
and the xy-plane.
c. Convert v = −12î + 9ĵ − 8k̂ to polar form, giving your answer to three decimal places.
~

THINK WRITE
a. 1. State the magnitude, r, and the two angles (𝜃, 𝜑). a.
~u = [10, 30°, 45°]
r = 10, 𝜃 = 30°, 𝜑 = 45°
2. Recall the formula x = r cos 𝜑 cos 𝜃 to resolve the x = r cos 𝜑 cos 𝜃
x-component. Substitute values (r, 𝜑 and 𝜃) and = 10 cos
( √(45°)
) (cos (30°)
√ )
evaluate. 2 3
= 10
2 2

5 6
=
2
3. Recall the formula y = r cos 𝜑 sin 𝜃 to resolve the y = r cos 𝜑 sin 𝜃
y -component. Substitute values (r, 𝜑 and 𝜃) and = 10 cos
( √(45°)
) (sin)(30°)
evaluate. 2 1
= 10
2 2

5 2
=
2
4. Recall the formula z = r sin 𝜑 to resolve the z = r sin 𝜑
z-component. Substitute values (r and 𝜑) and = 10 sin
(√ (45°)
)
evaluate. 2
= 10
2

=5 2
5. Conclude by representing the vector in component ~u = xî√+ yĵ + zk̂

form. 5 6 5 2 √
= î+ ĵ + 5 2k̂
2 2
b. 1. State the coefficient ( x, y, z) of each component b.
~v = −12î + 9ĵ − 8k̂
of the vector. ∴ x = −12, y = 9, z = −8

2. Calculate the magnitude of ~v. In polar form, r = (−12)2 + (9)2 + (−8)2
|~v| = r. = 17

CHAPTER 2 Vectors in three dimensions 39


“c02VectorsInThreeDimensions_print” — 2019/8/7 — 6:51 — page 40 — #19

z
3. Calculate the altitude angle by recalling the formula sin 𝜑 =
z r
sin 𝜑 = . Substitute values for ~r and 𝜑.
r −8
Solve for 𝜑 remembering −90° ≤ 𝜑 ≤ 90°. sin 𝜑 =
17
( )
−1 −8
𝜑 = sin
17
𝜑 ≈ −28.072°
The angle between ~v and the xy- plane
is −28.072°
y 9
c. 1. Recall the formula to calculate the azimuth angle, c. tan 𝜃 = =
y x −12
tan 𝜃 = . ( )
x −1 9
Substitute values for x and y. 𝜃 = tan
−12
𝜃 ≈ −36.870°
2. Determine in which quadrant 𝜃 lies. The angle is in the second quadrant as
As the x-value is negative and the y-value is x < 0 and y > 0
positive, the angle lies in the second quadrant. 𝜃 = 180° − 36.870°
Calculate the 2nd quadrant solution by subtracting 𝜃 = 143.130°
the magnitude of the angle from 180°.
3. State the polar equivalent for v.
~ ∴ ~v = −12î + 9ĵ − 8k̂ in polar form is
~v = [17, 143.130°, −28.072°]

Units 3 & 4 Area 2 Sequence 1 Concept 1


Introduction to vectors in three dimension Summary screen and practice questions

Exercise 2.2 Introduction to vectors in three dimensions


Technology free
1. State the coordinates of a point whose position vector is defined as:
a. ⃗
OA = 5î − 4ĵ b. ⃗OB = −2î + 6k̂
⃗ 5 ⃗
c. O C = î + 11ĵ − 12k̂ d. O D = −4î − ĵ + 8k̂
2
2. Determine the position vector of each of the following points relative to the origin, O:
a. A (5, 3, 2) b. B (2, 0, −6)
( )
2 2 22
c. C − , , − d. D (−11, 1, −4)
5 3 17
3. State if the following are either vector or scalar quantities.
a. Velocity
b. Volume
c. Force applied in a downward direction
d. Time
e. Speed

40 Jacaranda Maths Quest 12 Specialist Mathematics Units 3 & 4 for Queensland


“c02VectorsInThreeDimensions_print” — 2019/8/7 — 6:51 — page 41 — #20

4. Refer to the vectors, ~a = 4î − 2ĵ + 7k̂ and ~b = −3î + 5ĵ − k̂, to calculate:


a. −a b. 2b c. a + b d. 4~a − 2~b
~ ~ ~ ~
5. A rectangular prism (box) CDEFGHIJ can be defined by three vectors,
J I
~r, ~s and~t, as shown. F
E
Express in terms of ~r, ~s and ~t
t H
a. the vector joining C to E G ~
b. the vector joining C to I s
~ C r D
c. the vector joining D to J. ~

Technology active
6. An aeroplane travels 600 km east, then 900 km south. Calculate how far the aeroplane is from its starting
point and determine the true bearing of the resultant displacement.
7. WE1 If the point P has the coordinate (8, 4, 1), determine:

a. the vector ⃗OP b. the magnitude of ⃗ OP


c. the unit vector parallel to ⃗
OP .
8. If the point P has coordinates (6, 2, −8), determine:
a. the vector ⃗OP = p
~
b. the magnitude of the vector, |p|
~
c. the unit vector parallel to p
~
d. a vector 7 units long in the direction of p.
~
9. WE2 Three points, A, B and C have the coordinates (3, 4, 2), (8, 5, 10) and (9, 7, 11) respectively.

Calculate:
a. the vector ⃗AB b. the vector ⃗
BC

c. the vector sum u = 3 BC − 4 AB .⃗
~
10. WE3 The location of two points are defined by, P (1, 5, −6) and Q (9, 7, 4), respectively. Calculate the

position vector ⃗OM , where M is the midpoint of the line segment PQ.
11. Four points are given by P (−2, 5, 0), Q (0, 8, 4), R (3, −6, 12) and S (6, 17, 16). Refer to these points to
determine:
a. ⃗OP
b. ⃗PQ
c. c = 2 ⃗PQ − 3 ⃗ QR
~
d. the position vector ⃗OM , where M is the midpoint of the line segment PS
e. PQ and ⃗
if ⃗ QS are parallel.
12. WE4 For each of the following, examine the diagram and determine a · b.
a a
~ ~
a. ~ b. ~

6
3
𝜃 = 124°
b
𝜃 = 68° ~
3.5
b
7 ~

13. WE5 Given the vectors ~a = 2î − 5î + k̂ and ~b = −3î + 7ĵ − 4k̂, determine ~a · ~b.
14. WE6 The vectors given by, ~a = 3î + ĵ + 2k̂ and ~b = 4î + yĵ + 3k̂, are perpendicular. Determine the
value of y.

CHAPTER 2 Vectors in three dimensions 41


“c02VectorsInThreeDimensions_print” — 2019/8/7 — 6:51 — page 42 — #21

15. Let ~a = î − 2ĵ + 8k̂, ~b = 3î + 6ĵ − 9k̂ and ~c = 5î + 4ĵ − 2k̂. Demonstrate, using these vectors, the
property:
( )
~c · ~a − ~b = ~c · ~a − ~c · ~b
16. Calculate the angle between the two vectors, a = −5î + 6ĵ + 2k̂ and b = 7î + 11ĵ − 2k̂.
~ ~
17. WE7 Let ~a = 20î − 18ĵ + 8k̂ and ~b = −5î + 4ĵ − 2k̂. Determine:
a. the scalar resolute of b on a
~ ~
b. the vector resolute of b parallel to a, namely b∥
~ ~ ~
c. the vector resolute of b perpendicular to a, namely b⟂ .
~ ~ [ ]~
18. WE8a For each of the following, express u = r, 𝜃, 𝜑 in component form, xî + yĵ + zk̂.
~
a. u = [6, 61°, 32°] b. u = [20, 153°, −64°]
~ ~
c. u = [3, 120°, −40°] d. u = [15, 325°, 36°]
~ ~
19. a. WE8b Determine the altitude angle between the vector, u = xî + yĵ + zk̂ and the xy-plane in each
~
case below.
b. WE8c Convert u = xî + yĵ + zk̂ to polar form in each case below.
~
i.
~u = 2î + 5ĵ − 3k̂ ii. u = 3î − 4ĵ + 2k̂
~
iii. u = 5î + 6ĵ + 7k̂ iv. u = −4î + 5ĵ − 10k̂
~ ~
20. A boat travels 500 km in a direction S40°E and then
333 km in a southerly direction.
Calculate the resultant displacement from the starting
position giving both magnitude and direction as a
true bearing.
21. Let a = 6î + 2ĵ − k̂. Determine a vector parallel to a
~ ~
such that their dot product is −82.
22. An injured mountain climber is located at a
position relative to a camp given by the vector
~b = 150î + 200ĵ + 100k̂.
A rescue helicopter heads off from the camp in a
direction parallel to the vector ~a = 200î + 250ĵ + 125k̂.
All measurements are in metres.
a. Determine how far, in metres correct to two decimal
places, the rescue helicopter is from the camp when
closest to the mountain climber.
b. Calculate the minimum distance, in metres correct to
two decimal places, between the helicopter and the
mountain climber.

2.3 Geometric proofs using vectors


2.3.1 Geometrical shapes
Geometry is a branch of mathematics that explores the relationship between lines, shapes, solids and angles.
It is often referred to as deductive geometry because problems are solved by referring to mathematical
statements considered to be true, known as axioms or postulates. The most famous of which are Euclid’s
postulates, explored in Unit 1, Topic 3 (Chapter 4 of the Jacaranda Maths Quest 11 Specialist Mathematics
Units 1 & 2 for Queensland). Euclid’s first postulate stated that a straight line can be drawn between any
two points.

42 Jacaranda Maths Quest 12 Specialist Mathematics Units 3 & 4 for Queensland


“c02VectorsInThreeDimensions_print” — 2019/8/7 — 6:51 — page 43 — #22

Straight lines define the edges of many shapes and solids and can be expressed as directed line segments.
When considered in this way, vectors can be used to prove and solve many geometric problems. Consider
the common geometric shapes and their vector properties below.

Quadrilateral – General
A plane four-sided figure with no two sides necessarily parallel C
D
nor equal in length.

Trapezium B
A plane four-sided figure with one pair of sides parallel, but not equal. D C
In trapezium ABCD, since AB is parallel to DC, ⃗ AB = 𝜆 D ⃗C;
where 𝜆 is a scalar.

A B

Parallelogram
A plane four-sided figure with two sets of sides parallel and equal D C
in length.
In parallelogram ABCD, ⃗ AB = D ⃗ C and ⃗AD = ⃗ BC Also,
|⃗ |
AB = D | ⃗ |
C and ⃗| |
AD = ⃗ | |
BC .
A B

Rectangle
A parallelogram with all angles being 90°. D C
In rectangle ABCD, ⃗AB = D ⃗ C, ⃗AD = ⃗ BC and thus ⃗AB · ⃗ BC = 0,
⃗BC · C⃗D = 0, C⃗ D·⃗DA = 0 and ⃗ DA · ⃗AB = 0, since all these sides
are perpendicular.
A B

Rhombus
A parallelogram with all sides equal in length. D C
In rhombus ABCD, ⃗ AB = D ⃗C and ⃗AD = ⃗ BC
|⃗ |
AB = ⃗ | | |
BC = D ⃗ |
C = ⃗ | |
AD , since all sides are equal in length.

A B

CHAPTER 2 Vectors in three dimensions 43


“c02VectorsInThreeDimensions_print” — 2019/8/7 — 6:51 — page 44 — #23

Square
A rhombus with all angles 90°. D C
In square ABCD,
⃗AB = D ⃗ C, ⃗AD = ⃗ BC and
⃗AB · ⃗
BC = 0, ⃗ BC · C⃗D = 0, C⃗ D·⃗ DA = 0, ⃗
DA · ⃗
AB = 0, since all these sides
are perpendicular.
Also, | ⃗
AB | = | ⃗
BC | = | D
⃗ C| = |⃗
AD |

A B

WORKED EXAMPLE 9
( )
1
If O is the origin and U is the midpoint of the line segment TV, prove that ⃗
OU = ⃗ + OV
OT ⃗ .
2
T

O V

THINK WRITE
1. Define the vector ⃗
OV as the sum of the other two Let, ⃗
OV = ⃗
OT + ⃗
TV [1]
other vectors.
2. Rearrange [1] to make ⃗
TV as the subject. TV = ⃗
⃗ OV − ⃗
OT [2]
⃗ 1
3. Define the vector O U as the sum of ⃗ OT and ⃗ TU . O⃗U = ⃗
OT + ⃗TV [3]
1 2
As U is the midpoint of ⃗ TU = ⃗
TV , ⃗ TV .
1 (⃗ ⃗)
2
4. Substitute [2] into [3] and simplify the right-hand ⃗
O U =⃗OT + OV − OT
side. 2
1 (⃗ ⃗) ⃗ 1 1
Expand OV − OT . O U =⃗OT + ⃗ OV − ⃗ OT
2 2 2
Collect like vectors ⃗OT . 1 1
= ⃗ OT + ⃗ OV
Factorise to demonstrate the LHS = RHS. 2 2
1 (⃗ ⃗)
= OT + OV
2
⃗ 1 (⃗ ⃗)
5. Write a concluding statement. ∴ O U = OT + OV
2

44 Jacaranda Maths Quest 12 Specialist Mathematics Units 3 & 4 for Queensland


“c02VectorsInThreeDimensions_print” — 2019/8/7 — 6:51 — page 45 — #24

WORKED EXAMPLE 10
Prove that if the diagonals of a parallelogram are equal in length, then the parallelogram is a
rectangle.

C B

c
~

O a A
~

THINK WRITE
1. Define the vectors ~a and ~c in terms of Let O, A, B, C be a parallelogram.
vectors joining the points A, B, C, D. ⃗
~a = OA = ⃗
CB
State equivalent vectors using the
properties of a parallelogram. ~c =

O C =⃗ AB
2. Express the diagonals in terms of the ⃗
AC = O⃗ C−⃗ OA OB = ⃗
⃗ OA + ⃗ AB
vectors, ~a and ~c. = ~c − ~a = ~a + ~c

3. Assume the diagonals are equal in length. |⃗


OB | = | ⃗
AC |
| ⃗ |2 | ⃗ |2
4. Express the diagonals in terms of ~a and ~c (OB )= ( AC ) ( ) ( )
and square both sides to apply the dot ~a + ~c · ~a + ~c = ~c − ~a · ~c − ~a
product property, |~a|2 = ~a · ~a.

5. Expand both sides. Remember that LHS = ~a · ~a + ~a · ~c + ~c · ~a + ~c · ~c


2
~a · ~a = |~a| and ~a · ~c = ~c · ~a. = |~a|2 + 2~a · ~c + |~c|2
RHS = ~c · ~c + ~c · −~a + −~a · ~c + −~a · −~a
= |~c|2 − 2~a · ~c + |~a|2

6. Rewrite | ⃗
OB |2 = | ⃗
AC |2 in terms of the |~a|2 + 2~a · ~c + |~c|2 = |~c|2 − 2~a · ~c + |~a|2
dot product calculations and collect like 2~a · ~c + 2~a · ~c = |~c|2 + |~a|2 − |~a|2 − |~c|2
terms. 4~a · ~c = 0
~a · ~c = 0
7. State the conclusion. Since ~a · ~c = 0 this implies that ∠COA = 90°.
∴ ⃗OA is ⊥ to O ⃗ C , OABC is a rectangle.

This can extend to simple applications of plane shapes in three dimensions.

CHAPTER 2 Vectors in three dimensions 45


“c02VectorsInThreeDimensions_print” — 2019/8/7 — 6:51 — page 46 — #25

WORKED EXAMPLE 11
Given the points D (6, 1, − 3) , E (2, 3, − 4) and F (5, 7, − 8) , prove that DEF forms a
right-angled triangle at E, and hence calculate the area of the triangle as an exact value.

14

12

10
–14
8 –12
–10
6 –8
4 –6

–14 –4
–12 –10 2
–8 –6 –2
–4 –2 0 2 4 6 8 10
2 12 14
–2
4 y
6 –4
8 E (2, 3, –4)
–6
10
12 D (6, 1, –3) –8
14
–10
x
–12
F (5, 7, –8)
–14

THINK WRITE
1. Define each of the vertices as position D (6, 1, −3) ∴O⃗ D = 6î + ĵ − 3k̂
vectors. E (2, 3, −4) ⃗
∴ OE = 2î + 3ĵ − 4k̂
F (5, 7, −8) ∴⃗OF = 5î + 7ĵ − 8k̂
2. The line segments that form the triangle ⃗ ⃗ ⃗
DE = (OE − OD ) ( )
can be written as vectors in terms of = 2î + 3ĵ − 4k̂ − 6î + ĵ − 3k̂

O D, ⃗OE , ⃗
OF . = −4î + 2ĵ − k̂
EF = (⃗
⃗ OF − ⃗ OE ) ( )
= 5î + 7ĵ − 8k̂ − 2î + 3ĵ − 4k̂
= 3î + 4ĵ − 4k̂
( ) ( )
3. If ⃗
DE is perpendicular to ⃗DE · ⃗ EF = −4î + 2ĵ − k̂ · 3î + 4ĵ − 4k̂
⃗EF , then ⃗
DE · ⃗
EF = 0. = −12 + 8 + 4
=0
4. As the dot product is 0, the angle between Therefore ⃗ DE is ⊥ to ⃗
EF , hence the triangle is
the vectors is 90°. right-angled.

46 Jacaranda Maths Quest 12 Specialist Mathematics Units 3 & 4 for Queensland


“c02VectorsInThreeDimensions_print” — 2019/8/7 — 6:51 — page 47 — #26


5. Recall the area of a triangle can be |⃗
DE | =
(−4)2 + (2)2 + (−1)2
1 √
calculated by the rule, A = bh. = 21
2 √
In this case b = | ⃗
DE | and h = | ⃗
EF | |⃗ |
EF = (3)2 + (4)2 + (−4)2

= 41
1
6. Substitute the magnitude of the two vectors A = | ⃗DE || ⃗
EF |
into the area rule and state the result. 2
1√ √
= 21 × 41
2
1√
= 861 units2
2

WORKED EXAMPLE 12
A tetrahedron is defined by the vertices, O, L, M and N. N
Given that, ⃗
OL = ~l, ⃗
OM = m ⃗ n
~ and ON = ~
a. Express ⃗LM in terms of ~l, m
~ or ~n
b. Express ⃗MN in terms of ~l, m~ or ~
n
n
c. Express L⃗N in terms of ~l, m
~
~ or ~
n
d. Develop an expression for the line joining the midpoints
of OL to LN. m
~
O M

l
~
L

THINK WRITE
a. 1. Choose a path from L to M that travels along a. ⃗
L M = ⃗
LO + ⃗
OM
edges defined as vectors ~l, m ~ or ~n.
2. Recall that ⃗
LO = − ⃗ OL (negative of a vector). ⃗
L M =⃗
OM − ⃗OL
Express the RHS in terms of ~l, m ⃗ M =m −
3.
~ or ~n. L ~ ~ l
b. 1. Choose a path from M to N that travels along edges MN = ⃗
⃗ MO + O⃗N
defined as vectors ~l, m
~ or ~n.
2. Recall that MO = − ⃗
⃗ OM (negative of a vector). ⃗
MN = O⃗ N −⃗OM
Express the RHS in terms of ~l, m ⃗
MN = ~n − m
3.
~ or ~n. ~
c. 1. Choose a path from L to N that travels along edges LN = ⃗
⃗ LO + O ⃗N
defined as vectors ~l, m
~ or ~n.
2. Recall that ⃗
LO = − ⃗ OL (negative of a vector). ⃗
LN = O⃗ N−⃗ OL
Express the RHS in terms of ~l, m ⃗
LN = ~n − ~l
3.
~ or ~n.

CHAPTER 2 Vectors in three dimensions 47


“c02VectorsInThreeDimensions_print” — 2019/8/7 — 6:51 — page 48 — #27

d. 1. Define the midpoints of ⃗OL and ⃗ LN as P and Q Let P be the midpoint of OL.
respectively. Draw a sketch to assist visualising the Let Q be the midpoint of LN.
problem.
N

n
~
Q

m
~
O M

l
~ P

L ( )
⃗ 1 1
2. OP is parallel to ~l ⃗
OL and is half its magnitude. ⃗
OP = ⃗ OL = ~l
2 2
⃗ 1
3. O Q is the vector sum of ⃗
OL and one half of ⃗
LN . ⃗
O Q =⃗ OL + ⃗ LN
2

OL can be expressed as ~l. 1( )
= ~l + ~n − ~l

LN was defined as ~n − ~l. 2
Expand and simplify to then collect like vectors. 1 1
= ~l + ~n − l
2 2~
1 1
= ~l + ~n
2 2
4. ⃗
PQ can be determined by subtracting ⃗
OP from The vector joining P to Q is

O Q. ⃗ ⃗
PQ = O Q−⃗ OP
( ) ( )
1 1 1
= ~l + ~n − l
2 2 2~
1
= ~n
2

Units 3 & 4 Area 2 Sequence 1 Concept 2


Geometric proofs using vectors Summary screen and practice questions

Exercise 2.3 Geometric proofs using vectors


Technology free
1. WE9 OABC is a rhombus. The points P, Q, R and S are the midpoints of the sides OA, AB, BC and OC

respectively. Prove that PQRS is a rectangle.


2. WE10 Prove that if the diagonals of a parallelogram are perpendicular, the parallelogram is a rhombus.
3. WE11 Given the points A (−3, 5, 4), B (2, 3, 5) and C (4, 6, 1), prove that ABC forms a right-angled
triangle at B, and hence calculate the area of the triangle.

48 Jacaranda Maths Quest 12 Specialist Mathematics Units 3 & 4 for Queensland


“c02VectorsInThreeDimensions_print” — 2019/8/7 — 6:51 — page 49 — #28

4. Given the points A (4, 7, 3), B (8, 7, 1) and C (6, 5, 2):


a. Show that ABC forms an isosceles triangle.
b. Let M be the midpoint of AB, and show that MC is perpendicular to AB.
5. Prove Pythagoras’ theorem. y
6. a. Prove that the angle inscribed in a semicircle is a right angle.
b. The diagram shows a circle of radius r with centre at the origin O on C
the x- and y-axes. The points A and B lie on the diameter of the circle
ĵ
and are on the x-axis; their coordinates are (−r, 0) and (r, 0)
A O î B x
respectively. The point C has coordinates (a, b) and lies on the
circle, where a, b and r are all positive real constants. Show
that CA is perpendicular to CB.
7. Use vectors to prove that the line joining the midpoints of two sides of
a triangle is parallel to the third side and half its length.
8. OABC is a square. The points P, Q, R and S are the midpoints of the
sides OA, AB, BC and OC respectively. Prove that PQRS is a square.
9. Prove that the line segments joining the midpoints of consecutive sides of a rectangle form a rhombus.
10. AB and CD are two diameters of a circle with centre O. Letting ⃗ OA = ~a and O ⃗C = ~c, prove that
ACBD is a rectangle.
11. A median is formed inside a triangle when the midpoint of a side is joined to its opposite vertex. Prove
that medians inside a triangle sum to zero.
12. Prove that the vector ~ â + ~b bisects the angle between the vectors ~a and ~b.
13. Four points, ABCD, form a quadrilateral. Prove that the midpoints of the sides of the quadrilateral form
a parallelogram.
14. A parallelogram is defined by the vectors ~a and ~b. Prove the parallelogram law that states,
|~a + ~b|2 + |~a − ~b|2 = 2|~a|2 + 2|~b|2 .
WE12 A tetrahedron is defined by the vertices, O, L, M and N. Given that, ⃗ OL = ~l, ⃗
OM = m
15.
~ and

O N = ~n, show that the line segment formed between the midpoints of ⃗ LN to ⃗ ⃗
MN is parallel to the L M.
N

n
~

m
~
O M

l
~

L
16. A storage container is formed in the shape of a rectangular prism. Two support beams are being installed
on the inside of the container that would join O to E and G to C. Prove that the diagonals bisect one
another, if and only if, the dimensions of the container are a Pythagorean triad.
D
B
G
E

O C
O
A F

CHAPTER 2 Vectors in three dimensions 49


“c02VectorsInThreeDimensions_print” — 2019/8/7 — 6:51 — page 50 — #29

2.4 Cartesian and parametric equations


2.4.1 Cartesian coordinates for three-dimensional space
The three-dimensional coordinate system was introduced at the beginning z
of this chapter. The position of a point in three-dimensional space can
be defined in terms of the unit vectors î, ĵ and k̂ and the point P can be written in
the form of an ordered triple, P (x, y, z). Three-dimensional axes are commonly
drawn as a right-handed system - as the fingers on your right-hand curl from
the x-axis to the y-axis, your thumb points in the direction of the positive z-axis.

WORKED EXAMPLE 13
Refer to the three-dimensional space below to state the coordinates of A, B and C.

4 –7
–6
–7 3
–6 –5
–5 –4
–4 2 A
–3
–3
–2
–2 1
–1 –1

0 1
C 1
2 2
–1 3
3 4
4 –2 5
5 6
6 7
–3
7 B y
–4
x
–5

–6

–7

50 Jacaranda Maths Quest 12 Specialist Mathematics Units 3 & 4 for Queensland


“c02VectorsInThreeDimensions_print” — 2019/8/7 — 6:51 — page 51 — #30

THINK WRITE
1. Point A is 3 units in the negative direction of the Point A is defined as, A (−3, 4, 2)
x-axis, 4 units in the positive direction of the y-axis
and 2 units in the positive direction of the z-axis.
2. Point B is 2 units in the positive direction of the Point B is defined as, B (2, 2, −2)
x-axis, 2 units in the positive direction of the y-axis
and 2 units in the negative direction of the z-axis.
3. Point C does not have an x-axis component, but is Point C is defined as, C (0, −4, −2)
4 units in the negative direction of the y-axis and
2 units in the negative direction of the z-axis.

2.4.2 Distances between points in three-dimensions


Many of the formulae previously studied can be applied z
A (x1, y1, z1)
to three-dimensional Cartesian systems.
The distance between two points in three-dimensional
space, A (x1 , y1 , z1 ) and B (x2 , y2 , z2 ), can be calculated
using Pythagoras’s theorem twice to produce the formula
for the distance between two points.
B (x2, y2, z2)
y

The distance between two points A (x1 , y1 , z1 ) and B (x2 , y2 , z2 )



d = (x2 − x1 )2 + (y2 − y1 )2 + (z2 − z1 )2

Like midpoint calculations in two-dimensional space, the midpoint MAB of a line segment defined by the
Cartesian coordinates A (x1 , y1 , z1 ) and B (x2 , y2 , z2 ) can be found by taking the average of each of the
coordinates.

The midpoint of the line segment joining A (x1 , y1 , z1 ) and B (x2 , y2 , z2 )


( )
x1 + x2 y1 + y2 z1 + z2
MAB = , ,
2 2 2

WORKED EXAMPLE 14
A line segment is formed between two points, A (−2, 5, 8) and B (−10, 11 − 4).
a. Calculate the exact distance between A and B.
b. Determine the midpoint of the line segment AB.

CHAPTER 2 Vectors in three dimensions 51


“c02VectorsInThreeDimensions_print” — 2019/8/7 — 6:51 — page 52 — #31

THINK WRITE

a. 1. Recall the distance formula for two points d= (x2 − x1 )2 + (y2 − y1 )2 + (z2 − z1 )2
in three dimensional space.

2. Substitute the x, y, z coordinates into the d= (−10 − −2)2 + (11 − 5)2 + (−4 − 8)2

formula for points A and B. d = 64 + 36 + 144

3. Express the distance in simplified surd d = 244

form. d=2 ( 61 units )
x1 + x2 y1 + y2 z1 + z2
b. 1. Recall the midpoint formula for three MAB = , ,
dimensions. 2 2 2
( )
−2 + −10 5 + 11 8 + −4
2. Substitute the x, y, z coordinates into the = , ,
formula for points A and B. 2 2 2
3. Express the midpoint, MAB as coordinates MAB = (−6, 8, 2)
(x, y, z).

TI | THINK WRITE CASIO | THINK WRITE


a. 1. On a Calculator page, a. 1. On a Run-Matrix screen,
select select
MENU MAT/VCT
7: Matrix Vector Define Vectors A and B
7: Norms as shown in Worked
1: Norm example 1b.
Press the ENTER
button.

2. Complete
[ the entry
] line 2. On a Run-Matrix screen,
([ −10, 11,
] −4 − select
−2, 5, 8 ) OPTN
MAT/VCT
⊳ (x3 times)
Norm(

3. Press the ENTER 3. Complete the entry


button. line as
The answer appears on Vct ALPHA
the screen. X, 𝜃, T-Vct
ALPHA
Log)

4. Press the EXE button.


The answer appears on
the screen

52 Jacaranda Maths Quest 12 Specialist Mathematics Units 3 & 4 for Queensland


“c02VectorsInThreeDimensions_print” — 2019/8/7 — 6:51 — page 53 — #32

2.4.3 The Cartesian equation of a sphere y


P (x, y, z)
The sphere can be defined, in formal terms, as the
locus of points P (x, y, z) that is equidistant from a given
point C, where C is, C (xc , yc , zc ). More simply, a sphere is r C
simply the set of all points P (x, y, z) that are an equal
distance from the centre C (xc , yc , zc ).
The equation of a sphere, with radius r and centre
C (xc , yc , zc ), can be determined by applying the distance

formula d = (x2 − x1 )2 + (y2 − y1 )2 + (z2 − z1 )2
where the distance d is the radius of the sphere, r. O
z

If P (x, y, z) is a point on the sphere, the equation for a sphere is

r2 = (x − xc )2 + (y − yc )2 + (z − zc )2

WORKED EXAMPLE 15
a. Determine the Cartesian equation for the sphere with centre C (−3, 7, 2) and radius r = 9.
b. Determine the Cartesian equation for the sphere that has A (6, 12, 11) and B (−2, 4, −1) as the
endpoints of its diameter.
c. Determine the centre and radius of the sphere with the Cartesian equation
x2 + y2 + z2 − 12x + 4y + 8z + 20 = 0

THINK WRITE
a. 1. Identify the coordinates of the a. C (xc , yc , zc ) = C (−3, 7, 2)
centre of the circle. Express these ∴ xc = −3, yc = 7, zc = 2
values in terms of the variables
xc , yc , zc .
2. Square the given radius so that it Radius, r = 9
can be expressed as r2 . ∴ r2 = 92 = 81
Recall the general form for the
3. r2 = (x − xc )2 + (y − yc )2 + (z − zc )2
equation of a sphere and 81 = (x + 3)2 + (y − 7)2 + (z − 2)2
substitute in the values.
b. 1. State the given points, which b. A (6, 12, 11) and B (−2, 4, −1)
define the endpoints of a
diameter. ( )
x1 + x2 y1 + y2 z1 + z2
2. The midpoint, MAB , defines the MAB = , ,
2 2 2
center of the circle. Recall the ( )
rule for the midpoint and 6 + −2 12 + 4 11 + −1
= , ,
substitute the coordinates for 2 2 2
points A and B. = (2, 8, 5)
∴ The centre is given by C (2, 8, 5)

CHAPTER 2 Vectors in three dimensions 53


“c02VectorsInThreeDimensions_print” — 2019/8/5 — 19:21 — page 54 — #33

3. The centre of the circle defines C (xc , yc , zc ) = C (2, 8, 5)


the value for the associated xc = 2, yc = 8, zc = 5
variables xc , yc , zc .
4. The radius of a sphere, r, is the Calculate the distance from the centre to one of the
distance from the centre to any endpoints
√ of the diameter.
point on its surface. The radius
r= (x − xc )2 + (y − yc )2 + (z − zc )2
can then be defined by calculating √
the distance from the centre r= (6 − 2)2 + (12 − 8)2 + (11 − 5)2

C (2, 8, 5) to the point r = 16 + 16 + 36
A (6, 12, 11). √
r = 68
r2 = 68 units
5. Recall the general form for the The Cartesian equation of the sphere is
equation of a sphere and r2 = (x − xc )2 + (y − yc )2 + (z − zc )2
substitute in the values. 68 = (x − 2)2 + (y − 8)2 + (z − 5)2
c. 1. Complete the square on each of c. x2 + y2 + z2 − 12x + 4y + 8z + 20 = 0
the x−, y− and z− components to x2 − 12x + y2 + 4y + z2 + 8z + 20 = 0
rearrange the equation into the
x2 − 12x + y2 + 4y + z2 + 8z = −20
general form of a sphere r2 = ( )2 ( )2 ( )2
(x − xc )2 + (y − yc )2 + (z − zc )2 . 2
x − 12x +
−12 2
+ y + 4y+
4 2
+ z + 8z+
8
Remember to add any values 2 2 2
required to completed the square ( )2 ( )2 ( )2
−12 4 8
to both side of the equation. =−20 + + +
2 2 2
2 2 2
x − 12x + 36 + y + 4y + 4 + z + 8z + 16
=−20 + 36 + 4 + 16
2. The centre of the circle can be (x − 6)2 + (y + 2)2 + (z + 4)2 = 36
recognised from the equation (x − 6)2 + (y − (−2))2 + (z − (−4))2 = 36
generated. ∴ C (6, −2, −4)
3. To determine the radius, recall the The centre is (6, −2, −4) with a radius of 6.
general form of the sphere r2 = 36

r2 = (x−xc )2 +(y−yc )2 +(z−zc )2 . r = 36
=6

2.4.4 Parametric equations in two-dimensions


A function defines a dependent variable, y
20
y, in terms of an independent variable x. These
types of equations are in the form y = f (x) and have
15
graphs that are intersected only once by a vertical line. y = –0.01(x – 10)3 + 7
That is, for every x-value there is a unique y-value. 10
Consider the Cartesian equation,
y = −0.01 (x − 10)3 + 7 shown graphically 5
in the diagram. In this case, the function could model
the position of a particle travelling along a water slide. 0 x
5 10 15 20
The function can then define the point (x, y), where
the position of the particle is as a function of x.

54 Jacaranda Maths Quest 12 Specialist Mathematics Units 3 & 4 for Queensland


“c02VectorsInThreeDimensions_print” — 2019/8/7 — 6:51 — page 55 — #34

The table exhibits this relationship as ordered pairs.

x 0 5 10 15
y = f (x) 17 8.25 7 5.75

While the path of the particle can be described by the Cartesian equation, y = −0.01 (x − 10)3 + 7, it
would also be useful to know when the particle reaches a point location. This is achieved by introducing time
as a third variable. Here, time is referred to as the parameter.
Written as a parametric equation, the location of the particle on the slide can be defined in terms of the
parameter, that is:

x = t, y = −0.01(t − 10)3 + 7

For each number t, there are corresponding values of x and y (and hence corresponding positions), shown
in the table below.

t 0 1 5 10 18
x=t 0 1 5 10 18
y = −0.01 (t − 10)3 + 7 17 14.29 8.25 7 1.88
(x, y) (0, 17) (1, 14.29) (5, 8.25) (10, 7) (18, 1.88)

Any letter can be used to represent the parameter. While the letter t is commonly used, it does not have to
be defined as time. Where equations of curves are being examined, the angle 𝜃 is often the parameter.

If f (t) and g (t) are two continuous functions defined by the parameter t, then the equations
x = f (t) and y = g (t) are referred to as parametric equations. The collection of points (x, y)
obtained as t varies is called a plane curve.

Simple algebraic techniques can be used to convert a set of parametric equations into Cartesian form (and
vice versa). The following examples will focus on parametric equations in two dimensions.

WORKED EXAMPLE 16
Determine the equivalent Cartesian equation for the following parametric equations.
a. x = 2t, y = t + 3 b. x = t − 3, y = 2t2
2
2t 1−t
c. x = , y=
2
1+t 1 + t2
THINK WRITE
a. 1. Define each parametric equation as equation [1] a. x = 2t [1]
and [2]. y=t+3 [2]
2. To convert to a Cartesian equation, the parameter t x = 2t
needs to be eliminated. Solve for t in equation [1]. x
t=
2
3. Substitute the solution for t into [2]. That is, t is y=t+3 [2]
being replaced with x. x
y= +3
The Cartesian equation represents the graph of 2
a straight line.

CHAPTER 2 Vectors in three dimensions 55


“c02VectorsInThreeDimensions_print” — 2019/8/7 — 6:51 — page 56 — #35

b. 1. Define each parametric equation as equation [1] b. x=t−3 [1]


and [2]. y = 2t2
[2]
2. The parameter t needs to be eliminated. Solve for x=t−3
t in [1]. t=x+3
3. Substitute the solution for t into [2]. That is, t is y = 2t2 [2]
2
being replaced with x. y = 2 (x + 3)
The Cartesian equation represents the graph of y = 2(x2 + 6x + 9)
a parabola.
y = 2x2 + 12x + 18
2t
c. 1. Define each parametric equation as equation [1] c. x = [1]
and [2]. 1 + t2
1 − t2
y= [2]
1 + t2
( )2
2 2t
2. As the parameter t is not easily eliminated, Squaring [1] gives, x =
a different approach is needed. 1 + t2
( )2
Try squaring both sides of equation [1] and [2]. 2 1 − t2
Squaring [2] gives, y =
1 + t2
( )2 ( )2
2 2 2t 1 − t2
3. Add the equations, [1] + [2]. x +y = +
1 + t2 ( 1 +2 )t2
2

(2t)2 1−t
4. Rewrite both fractions as the square of the =( ) 2
+ ( )2
numerators and denominators. Add the fractions 1 + t2 1 + t2
as they have like denominators. 4t2 + 1 − 2t2 + t4
= ( )2
1 + t2
t4 + 2t2 + 1
= ( )2
1 + t2
(2 )2
t +1
5. The numerator (can be)factorised as =( )2
2
t4 + 2t2 + 1 = t2 + 1 using the substitution t2 + 1
∝ = t2 . =1
6. Cancel like terms and state the relation. In this case ∴ x2 + y2 = 1
it is a circle of radius 1.

Parametric equations are useful when describing the curve y


of complex graphs and relations. In Worked example 16c, the
parametric equation described a circle. If the equation of 1 y = √1 – x2
a circle was written in the form y = f (x), it would be

y = 1 − x2 . When graphed, it produces the top half of
a circle. The bottom half of the circle could be expressed

as y = − 1 − x2 .
Curves such as circles and ellipses can be expressed x
–1 0 1
in parametric form when the angle 𝜃 or A is the parameter.
When parametric equations involve trigonometric functions,
we often need to make use of standard trigonometric identities to eliminate the parameter. Recalling the
Pythagorean identities, addition identities, double-angle identities and factorisation identities will help
calculate and solve Cartesian equations of parametric equations.

56 Jacaranda Maths Quest 12 Specialist Mathematics Units 3 & 4 for Queensland


“c02VectorsInThreeDimensions_print” — 2019/8/7 — 6:51 — page 57 — #36

1. The Pythagorean identities 2. The addition identities


sin2 (A) + cos2 (A) = 1 sin (A ± B) = sin (A) cos (B) ± cos (A) sin (B)
sec2 (A) = 1 + tan2 (A) cos (A ± B) = cos (A) cos (B) ± sin (A) sin (B)
tan (A) ± tan (B)
cosec2 (A) = 1 + cot2 (A) tan (A ± B) =
1 ± tan (A) tan (B)
3. The double-angle identities 4. Angle sum and angle difference identities
sin (2A) = 2 sin (A) cos (A) sin (A + B) = sin (A) cos (B) + cos (A) sin (B)
cos (2A) = cos2 (A) − sin2 (A) sin (A − B) = sin (A) cos (B) − cos (A) sin (B)
cos (A + B) = cos (A) cos (B) − sin (A) cos (B)
cos (2A) = 1 − 2 sin2 (A) cos (A − B) = cos (A) cos (B) + sin (A) cos (B)
2 tan (A)
tan (2A) =
1 − tan2 (A)
5. Multiple-angle identities 6. Product identities
2 1
sin (3A) = 2 sin (A) − 4 sin (A) sin (A) sin (B) = (cos (A − B) − cos (A + B))
2
cos (3A) = 4 cos2 (A) − 3 cos (A) 1
cos (A) cos (B) = (cos (A − B) + cos (A + B))
sin (4A) = cos (A)(4 sin(A) − 8 sin2 (A)) 2
cos (4A) = 8 cos4 (A) − 8 cos2 (A) + 1 1
sin (A) cos (B) = (sin (A + B) + sin (A − B))
2
1
cos (A) sin (B) = (sin (A + B) − sin (A − B))
2
7. Half-angle identities
( ) ( )
A A
sin (A) = 2 sin cos
2 2
( ) ( )
A A
cos (A) = cos2 − sin2
2 2
( )
A
= 2 cos2 −1
2
( )
2 A
= 1 − 2 sin
2
WORKED EXAMPLE 17
Calculate the Cartesian equation for the following parametric equations:
a. x = cos 𝜃, y = sin 𝜃 b. x = 3 cos 𝜃, y = 5 sin 𝜃
c. x = 2 sec 𝜃, y = 3 tan 𝜃 d. x = sin 𝜃, y = cos 2𝜃

THINK WRITE
a. 1. Define each parametric equation as equation [1] a. x = cos 𝜃 [1]
and [2]. y = sin 𝜃 [2]
2. Square equations [1] and [2] to allow the x2 = cos2 𝜃 [3]
Pythagorean identities to be used, and define new y2 = sin2 𝜃 [4]
equations.
3. Add equations [3] and [4]. x2 + y2 = cos2 𝜃 + sin2 𝜃 [3] + [4]

CHAPTER 2 Vectors in three dimensions 57


“c02VectorsInThreeDimensions_print” — 2019/8/7 — 6:51 — page 58 — #37

4. Substitute the Pythagorean identity x2 + y2 = 1


sin2 (A) + cos2 (A) = 1. The result is the Cartesian
equation of a circle.
b. 1. Define each parametric equation as equation [1] b. x = 3 cos 𝜃 [1]
and [2]. y = 5 sin 𝜃 [2]

2. Begin by squaring equations [1] and [2]. Rewrite the x2 = 9 cos2 𝜃


resulting equation in terms of either cos2 𝜃 or sin2 𝜃.
x2
= cos2 𝜃 [3]
9
y2 = 25 sin2 𝜃

y2
= sin2 𝜃 [4]
25
x2 y2
3. Add equations [3] and [4]. + = cos2 𝜃 +
9 25
sin2 𝜃 [3]+[4]
x2 y2
4. Substitute the Pythagorean identity + =1
sin2 (A) + cos2 (A) = 1. The result is the Cartesian 9 25
equation of an ellipse.
c. 1. Define each parametric equation as equation [1] c. x = 2 sec 𝜃 [1]
and [2] y = 3 tan 𝜃 [2]
x2
2. Square equation [1] and rearrange the result making x2 = 4 sec2 𝜃 ∴ sec2 𝜃 = [3]
sec2 𝜃 the subject. 4
y2
3. Square equation [2] and rearrange the result making y2 = 9 tan2 𝜃 ∴ tan2 𝜃 = [4]
tan2 𝜃 the subject. 9
x2 y2
4. Subtract equation [4] from [3], sec2 𝜃−tan2 𝜃 = − [3]−[4]
4 9
5. Substitute the Pythagorean identity sec2 𝜃 − tan2 𝜃 = 1
sec2 (A) − tan2 (A) = 1. The result is the Cartesian
equation of a hyperbola. x2 y2
− =1
4 9
d. 1. Define each parametric equation as equation [1] d. x = sin 𝜃 [1]
and [2] y = cos 2𝜃 [2]
2. Begin by squaring equation [1]. x2 = sin2 𝜃 [3]
3. Recall the double-angle identity cos 2𝜃 = 1 − 2 sin2 𝜃
cos (2A) = 1 − 2 sin2 (A). Substitute into equation y = 1 − 2x2
[2] then [3] to form the Cartesian equation.

2.4.5 Parametric equations as vector functions


Recall the vector location, by definition, of a point P (x, y), is given by the position vector ⃗
OP = xî + yĵ.
Now consider a vector where the x- and y- components are not constants but vary dependent on a parameter,
t. If we write ~r (t) to represent ⃗
OP , according to the parameter t, then the position of any particle can be
defined by a parametric equation.

58 Jacaranda Maths Quest 12 Specialist Mathematics Units 3 & 4 for Queensland


“c02VectorsInThreeDimensions_print” — 2019/8/7 — 6:51 — page 59 — #38

The position of any particle in two-dimensional space whose location varies according to
a parameter, t can be defined by
~r (t) = x (t) î + y (t) ĵ
where x (t) and y (t) are the respective horizontal and vertical components of the position vector at
any real value of the parameter t.

Equations written in this form are defined as a vector function. That is, the vector location of a particle is
a function of t. In many instances, the parameter t, is time.
Returning to the example of the path of a particle on a water slide defined by the parametric equations

x = t, y = −0.01 (t − 10)3 + 7.

The parametric equation can then be used to define the path of the same particle as a vector function.
( 3 )
r
~ (t) = (t) î + −0.01 (t − 10) + 7 ĵ

A sketch of the particle’s path can be depicted using vectors. For each new value of t, a new position
vector, ~r (t), is formed.

y
20

18
r (t) = (t) î + (–0.01(t – 10)3 + 7) ĵ
~
16

14 t=1

12
t=3
10
t=5
8 t = 10
6

2 t = 18

0 x
–2 2 4 6 8 10 12 14 16 18 20
–2

It is common to define the domain for a vector function. In the case above, the curve was graphed for the
interval, 0 ≤ t ≤ 18.88.

WORKED EXAMPLE 18
Let a particle’s
( position
) (metres), as a function of time (seconds), be given by
2
~r (t) = (t) î + −t + 6t ĵ, for 0 ≤ t ≤ 4.
a. Determine the particles location at t = 1 second.
b. Generate a table of values to model the position, r (t) = xî + yĵ, of the particle for integer
~
values of t.

CHAPTER 2 Vectors in three dimensions 59


“c02VectorsInThreeDimensions_print” — 2019/8/7 — 6:51 — page 60 — #39

THINK WRITE
a. Substitute t =(1 into the) equation a. For ~r (t), let t = 1.
2 ( 2 )
~r (t) = (t) î + −t + 6t ĵ to determine ~r (t) = (t) î + −t + 6t ĵ
the particle’s position at t = 1. ( 2 )
~r (1) = (1) î + − (1) + 6 × 1 ĵ
= î + 5ĵ
b. The process used to determine ~r (1) is repeated for b.
t x (t) y (t)
all valid integer values of t for the stated domain ~r (t)
0 ≤ t ≤ 4. Summarise the results in table form. 0 0 0 0î + 0ĵ

1 1 5 î + 5ĵ

2 2 8 2î + 8ĵ

3 3 9 3î + 9ĵ

4 4 8 4î + 8ĵ

WORKED EXAMPLE 19
( √)
The position of a particle is given by the vector function ~r (t) = 2 t î + (5t − 2) ĵ, for t ≥ 0.
a. Express the position of the particle as a Cartesian equation.
b. Calculate the domain and range of the equation, then graph the path of the particle on a
Cartesian plane.
c. At a certain point on the particles path, its location is r (t) = xî + 8ĵ. Use the vector function to
~
calculate the value for x.

THINK WRITE

a. 1. Define each parametric equation as equation [1] x=2 t [1]
and [2]. y = 5t − 2 [2]

2. The parameter t needs to be eliminated. Solve for x=2 t
t in the equation [1]. x √
= t
2
( )2
x
t=
2
x2
t= [3]
4
3. Substitute [3] into [2]. Substitute [3] into [2].
Simplify and state the Cartesian equation. x2
t= [3]
4
y = 5t(− 2 ) [2]
x2
y=5 −2
4
5x2
y= −2
4

60 Jacaranda Maths Quest 12 Specialist Mathematics Units 3 & 4 for Queensland


“c02VectorsInThreeDimensions_print” — 2019/8/7 — 6:51 — page 61 — #40

b. 1. As t ≥ 0 calculate the domain√and range by b. When t = 0


substituting t = 0 into x = 2 t and y = 5t − 2. √
x=2 t

=2 0
=0
y = 5t − 2
=5×0−2
= −2
As t ≥ 0, it follows from the
parametric equations that x ≥ 0 and
y ≥ −2.

2. Construct the graph of the Cartesian equation using y


12 2
suitable technology. y = 5x – 2
11 4
10
9
8
7
6
5
4
3
2
1
x
–2 –1 0 1 2 3 4 5 6 7 8
–1
–2 (0, –2)


c. 1. Equate the two position vectors. c. 2 t î + (5t − 2) ĵ = xî + 8ĵ
2. Equate the y–component of r(t) = xî + 8ĵ with the (5t − 2) ĵ = 8ĵ
known parametric equation y = (5t − 2) ĵ
3. Solve for t. 5t − 2 = 8
8+2
t=
5
t=2
4. Determine the value of x by evaluating the x î can be calculated
√ by substituting
parametric equation for when t = 2 t = 2 in x = 2 t

x=2 2

5. State the answer. x=2 2

Parametric equations as vector functions in three dimensions


The use of parameters can be extended to three-dimensional spaces. The vector function, ~r (t), would contain
the parametric equations,

x = f (t), y = g (t), z = h (t)

CHAPTER 2 Vectors in three dimensions 61


“c02VectorsInThreeDimensions_print” — 2019/8/7 — 6:51 — page 62 — #41

The position of any particle in three-dimensional space whose location varies according to
a parameter, t, would consist of three components

~r (t) = x (t) î + y (t) ĵ + z(t)k̂


where x (t), y (t), z (t) are the respective components of the position vector at any real value of the
parameter t.

The procedures studied in the two-dimensional examples, are applicable in three dimensions, as shown in
the following worked example.

WORKED EXAMPLE 20
( ) ( )
The position (metres) of a bird is given by ~r (t) = (2t) î + 0.01t3 ĵ + 0.5t2 k̂ for t ≥ 0 seconds.
a. Determine the position of the bird at t = 0 and t = 15 seconds.
b. Calculate the straight-line distance of the bird from the origin at t = 15 seconds, to two
decimal places.

THINK WRITE
a. 1. Substitute t = 0( into the
) equation
( ) a. Let t = 0. (
3 3) ( 2)
r
~ (t) = (2t) î + 0.01t ĵ + 0.5t2 k̂ to ~r (0) = (2 × 0) î + 0.01 (0) ĵ + 0.5 (0) k̂
determine the bird’s position at t = 0. = 0î + 0ĵ + 0k̂
Hence, at t = 0 the bird’s position is the origin.
2. Substitute t = 15( into3 )the equation
( ) Let t = 15. (
2 3) ( 2)
r
~ (t) = (2t) î + 0.01t ĵ + 0.5t k̂ to ~r (15) = (2 × 15) î + 0.01 (15) ĵ + 0.5 (15) k̂
determine the bird’s position at t = 15. = 30î + 33.75ĵ + 112.5k̂
Hence at t = 15, the bird’s position is
30î + 33.75ĵ + 112.5k̂.

b. 1. The distance of the bird from the origin at b. |r (15) | = x2 + y2 + z2
~ √
t = 15 is the magnitude of the position
= (30)2 + (33.75)2 + (112.5)2
vector at t = 15.
Recall the magnitude formula for = 121.22
three-dimensional
√ vectors
|~u| = x2 + y2 + z2 .
2. Conclude with a statement. After 15 seconds of flight the bird is 121.22
metres from the origin.

Units 3 & 4 Area 2 Sequence 1 Concept 3


Cartesian and parametric equations Summary screen and practice questions

62 Jacaranda Maths Quest 12 Specialist Mathematics Units 3 & 4 for Queensland


“c02VectorsInThreeDimensions_print” — 2019/8/7 — 6:51 — page 63 — #42

Exercise 2.4 Cartesian and parametric equations


Technology free
1. WE13 Refer to the three-dimensional space below to state the coordinates of the following points.
a. Point A b. Point B c. Point C d. Point D e. Point E

–6
–7 3
–6 –5
–5 –4
2
–4 –3
–3
–2
–2 1
C –1 –1

0 E
1 1
2
2
–1 3
B 3 4
4 –2 A D 5
5 6

6 –3 y
7
–4
x

2. WE14a A line segment is formed between two points, A (x1 , y1 , z1 ) and B (x2 , y2 , z2 ). Calculate the

exact distance between A and B.


a. A (−5, 8, 12) and B (7, 15, 21) b. A (7, −2, 9) and B (3, 1, −2)
c. A (5, 2, 1) and B (−3, 7, −11) d. A (1, −2, 7) and B (−9, 3, −2)
3. WE14b A line segment is formed between two points, A (x1 , y1 , z1 ) and B (x2 , y2 , z2 ). Determine the
midpoint of the line segment AB.
a. A (5, 2, −7) and B (9, 12, 5) b. A (−10, 2, −2) and B (4, 6, −12)
c. A (7, −2, 10) and B (1, −10, −2) d. A (2, 2, 0) and B (9, −6, −9)
4. WE15a Determine the Cartesian equation for the sphere with centre C (xc , yc , zc ) and radius r.
a. r = 6 and C (2, 8, −3) b. r = 3 and C (−3, 1, −9)

c. r = 7 and C (1, 0, 6) d. r = 11 and C (−9, −2, 7)
5. WE15b Determine the Cartesian equation for the sphere that has A (x1 , y1 , z1 ) and B (x2 , y2 , z2 ) as the
endpoints of its diameter.
a. A (7, 13, 8) and B (−1, 3, 2) b. A (−5, −3, −1) and B (3, 1, 11)
c. A (2, −4, 5) and B (−8, 0, −25) d. A (1, −4, 8) and B (9, −8, −2)

CHAPTER 2 Vectors in three dimensions 63


“c02VectorsInThreeDimensions_print” — 2019/8/7 — 6:51 — page 64 — #43

6. Express the following equations in the expanded form of a sphere, x2 + y2 + z2 + ax + by + cz + d = 0.


2 2 2
a. (x − 5) + (y − 7) + (z − 1) = 144
2 2 2
b. (x + 1) + (y − 3) + (z + 2) = 81
2 2
c. (x + 3) + (y − 1) + z2 = 15
2 2 2
d. (x − 1) + (y − 4) + (z + 3) = 56
7. WE15c Each of the following equations have been expressed in the expanded form of a sphere.
Determine the centre and radius of the following spheres.
a. x2 − 10x + y2 + 4y + z2 + 12z − 16 = 0
b. x2 + y2 + z2 + 2x + 8y − 16z = 0
c. x2 + y2 + z2 + 20x − 12y + 24z + 180 = 0
d. x2 + y2 + z2 + 2x + 4y + 6z − 11 = 0
8. WE16 Determine the equivalent Cartesian equation for the following parametric equations.
a. x = 3t + 2, y = 2t + 5
b. x = t − 5, y = −t2 + t
c. x = 2t − 1, y = 4t2 + 7
t2
d. x = t3 , y =
4
1
e. x = 4 + , y = 4 − t
t
2 t
f. x = √ ,y =
t+1 t + 1
9. WE17 Calculate the Cartesian equation for the following parametric equations.
a. x = sin 𝜃, y = cos 𝜃 b. x = cos 3𝜃, y = sin 3𝜃
c. x = 4 cos 𝜃, y = 7 sin 𝜃 d. x = cosec 𝜃, y = cot 𝜃
10. Determine the equivalent Cartesian equation for the following vector functions.
t ( )
a. r (t) = 5t î + ĵ b. r (t) = 4t î + t2 − t ĵ
~ ~
(3 )
c. r (t) = 3t î + t3 + 2 ĵ d. r (t) = (2 cos t) î + (2 sin t) ĵ
~ ~
11. WE18 A particle’s position (metres), as a function of time (seconds), is given by
( 2
)
r
~ (t) = (t) î + −0.5t + 4t ĵ, for 0 ≤ t ≤ 4.
a. Determine the particle’s location at t = 1 second.
b. Generate a table of integer values to model the position, r (t) = x î + y ĵ, of the particle for integer
~
values of t.
Technology active
12. Let a particle’s position (metres), as a function of time (seconds), is given by
(2 )
r
~ (t) = (2t) î + t − 8t + 16 ĵ, for 0 ≤ t ≤ 5.
a. Determine the particle’s location at t = 3 s.
b. Generate a table of integer values to model the position of the object for integer values of t.
c. Calculate a Cartesian equation and determine the domain and range.
d. Graph the position of the particle on a Cartesian plane.
(√ )
t
13. WE19 The position of a particle is given by the vector function ~r (t) = 3 î + (2t + 7) ĵ, for t ≥ 0.
4
a. Express the position of the particle as a Cartesian equation.
b. Calculate the domain and range of the equation, then graph the path of the particle on a
Cartesian plane.
c. At a certain point on the particle’s path, its location is r (t) = x î + 71 ĵ. Use the vector function to
~
calculate the value for x.

64 Jacaranda Maths Quest 12 Specialist Mathematics Units 3 & 4 for Queensland


“c02VectorsInThreeDimensions_print” — 2019/8/7 — 6:51 — page 65 — #44

14. The position of an object is defined by the vector function, ~r (t) = (3 − 5t) î + (2 + 3t) ĵ + (t + 1) k̂,
for t ∈ R.
a. Generate a table of values for the position of the object at t = 1, t = 3 and t = 5.
b. Sketch the graph of the vector function to describe any interesting features.
15. The position of two particles are defined by the following equations,

~r a (t) = 5 sin(t) î + 5 cos(t) ĵ + 10k̂


~r b (t) = 6 sin(t) î + 6 cos(t) ĵ − 2k̂

a. Use appropriate technology to sketch the position of both particles.


b. Compare the two graphs to describe any similarities or differences. ( ) ( )
16. WE20 The position (metres) of a bird is given by r (t) = (3t) î + 0.02t3 ĵ + 0.4t2 k̂ for t ≥ 0 seconds.
~
a. Determine the position of the bird at t = 0 and t = 12 seconds.
b. Calculate the straight-line distance of the bird from the origin at t = 12 seconds.
17. A paragliding enthusiast( begins their)flight( from a cliff face.
) The path of the paraglider is given by
2 3
r
~ (t) = (850 − 15t) î + 950 − 0.4t ĵ + 350 − 0.01t k̂ for 0 ≤ t ≤ b. The pilot aims to finish the
flight somewhere near the airport, defined as the point O (0, 0, 0).
a. Calculate the initial position of the paraglider.
b. Determine the domain by calculating the largest
possible value for b. State any assumptions made in
developing a solution.
c. Determine the ground level distance the pilot is from
the airport when the paraglider lands.
d. Analyse the flight path of the paraglider and determine if it at
any point it ascended in altitude.
18. For each of the following vector equations, use technology to sketch the equation of the path.
a. x = cos(4t), y = sin(6t) for t ≥ 0
b. x = sin(5t), y = cos(t) for t ≥ 0
c. x = −cos(2t), y = sin(3t) for t ≥ 0
19. For each of the following vector equations, use technology to sketch the equation of the path.
a. The Spiral: r (t) = t cos(t) î + t sin(t) ĵ, for 0 ≤ t ≤ 6𝜋
~
b. The Cycloid: r (t) = 4 (t − sin(t)) î + 4 (1 − cos(t)) ĵ, for −2𝜋 ≤ t ≤ 4𝜋
~
c. The Witch of Agnesi: ~r (t) = 2 cot(t) î + 2 sin2 (t) ĵ, for −𝜋 ≤ t ≤ 𝜋
d. Lissajous curve: r (t) = 3 cos(t) î + 2 sin(2t) ĵ, for −𝜋 ≤ t ≤ 𝜋
~
3t 3t2
20. The path of particle is defined by the vector function, ~r (t) = î + ĵ, for −10 ≤ t ≤ 10.
1 + t3 1 + t3
Show that the Cartesian equation is given by x3 + y3 − 3xy = 0

2.5 The vector equation of a straight line


In your studies thus far, the equation of a line has been considered only in terms of a scalar equation in the
form y = mx + c. Equations in this form describe the straight-line relationship existing between two
variables (x, y). We will now explore how position vectors can be used to define a straight line in both two
and three dimensions.

CHAPTER 2 Vectors in three dimensions 65


“c02VectorsInThreeDimensions_print” — 2019/8/7 — 6:51 — page 66 — #45

2.5.1 Lines in two dimensions y


Consider the two-dimensional plane containing
the points A (−1, 1) and B (3, 3). The location 4
of these points can be expressed as position vectors: B
3
• ⃗
OA = ~a = −1î + 1ĵ
2

• ⃗
OB = ~b = 3î + 3ĵ A b
~
1
a
~
0 O x
–3 –2 –1 1 2 3 4
–1

Similarly, the location of the vector connecting y


the two points is defined as
4


AB = ~b − ~a = ~d = 4î + 2ĵ 3 B
a+d
~ ~
The line segment joining points A and B can now 2
be described with the two vectors, ~a and ~d. b
A ~
• The ‘starting point’ is the head of position vector 1 d
~
~a = −1î + 1ĵ
a
~
• The line segment is formed by travelling (adding) –3 –2 –1 0 O 1 2 3 4
x
in the direction of ~d = 4î + 2ĵ.
–1

Vectors can also be used to define a line in both two y


and three dimensions. In this example a line can be
4 P (x, y, z)
formed by extending the line segment connecting
A and B. B
3
The position of any point P (x, y, z) on this a+d
~ ~
line can be calculated by adding scalar multiples 2
of ~d to the initial position vector ~a. A b
1 ~
a
~
0 O x
–3 –2 –1 1 2 3 4
–1

2.5.2 The vector equation of a straight line in two dimensions


The vector equation of a line is given by ~r = ~a + k ~d.
Where:
• ~r is the position vector of any point on the line
• ~a is the position vector of a known point on the line
• ~d is the vector travelling in the same direction (parallel) to the line
• the varying (scalar) parameter is k, where k ∈ R.

66 Jacaranda Maths Quest 12 Specialist Mathematics Units 3 & 4 for Queensland


“c02VectorsInThreeDimensions_print” — 2019/8/5 — 19:21 — page 67 — #46

In changing the parameter k, the vector ~r can have its head at any point on the line. Consider again the
two-dimensional plane containing the points A (−1, 1) and B (3, 3).
• The line between points A and B is defined by the vector equation, ~r = (−1î + 1ĵ) + k (4î + 2ĵ).
• Two points on the line will be located by varying the parameter k.
Scenario 1: k = 1.3 y

~r = (−1î + 1ĵ) + 1.3 (4î + 2ĵ) 4 P1(4.2, 3.6)


= −1î + 1ĵ + 5.2î + 2.6ĵ B
= 4.2î + 3.6ĵ 3
a+d
~ ~
∴ P1 (x, y) = (4.2, 3.6) 2
A b
1 ~ d
~
a
~
0 O x
–3 –2 –1 1 2 3 4
–1

Scenario 2: k = −0.2 y

~r = (−1î + 1ĵ) − 0.2 (4î + 2ĵ) 4


= −1î + 1ĵ − 0.8î − 0.4ĵ B
= −1.8î + 0.6ĵ 3
a+d
~ ~
∴ P2 (x, y) = (−1.8, 0.6) 2
A b
1 ~ d
P2(–1.8, 0.6) ~
a
~
0 O x
–3 –2 –1 1 2 3 4
–1

WORKED EXAMPLE 21
a. Determine a vector equation, in the two-dimensional space (x, y), for the line passing through
the points A (−1, 7) and B (4, −2).
b. Calculate the vector location of the point r on the line if the parameter k = 2.
~

THINK WRITE
a. 1. Define the position vectors ~a and ~b. a. ⃗
OA = ~a = −1î + 7ĵ

OB = ~b = 4î − 2ĵ
2. Calculate the direction vector, ~d, between points A ~d = ~b − ~a
and B. This vector is parallel to the line.
~d = (4î − 2ĵ) − (−1î + 7ĵ)
~d = 5î − 9ĵ
3. Recall the formula for the vector equation of a line ~r = ~a + k~d
and substitute in the expressions for ~d and ~a. The
~r = (−î + 7ĵ) + k (5î − 9ĵ)
vector equation of the line has been determined.

CHAPTER 2 Vectors in three dimensions 67


“c02VectorsInThreeDimensions_print” — 2019/8/7 — 6:51 — page 68 — #47

b. The location of ~r can be calculated by substituting b. Let k = 2


k = 2 into the vector equation. The solution is ~ = (−î + 7ĵ) + 2 (5î − 9ĵ)
r
expressed as a position vector. = (−î + 7ĵ) + (10î − 18ĵ)
= 9î − 11ĵ

2.5.3 Lines in three dimensions


Vector equations can be developed in three-
z
dimensional space providing (we have: )
• the position vector ~a = x1 î, y1 ĵ, z1 k̂ A = (–3, 6, 5)
that corresponds to a(point on the )line and,
• second vector ~u = x2 î, y2 ĵ, z2 k̂
a
that is a vector in the direction of the line.
Alternatively,
• the equation of a line can be defined if y
two points (A and B) on the same a
~
line are known, as shown in the diagram.
As for two-dimensional space, the vector
equation ~r = ~a + k ~d will define the
location of any point P (x, y, z) in a three- b
O ~
dimensional space, in most instances. B = (5, 2, 1)
Consider any point P (x, y, z) which lies along a
line and can be located in terms of ~a and ~d, where: P (x, y, z)
• ~d is the position vector between the
x
points A and B.
• The line containing points A and B and
runs in the same direction as ~d. d
~
• The position of any point P (x, y, z) can be
determined by starting at ~a and then travelling
along the line in scalar (k) products of ~d.
• Hence, the formula is given by ~r = ~a + k ~d.

WORKED EXAMPLE 22
a. Determine a vector equation, in the three-dimensional space (x, y, z), for the line passing
through the points A (−3, 6, 5) and B (5, 2, 1).
b. Verify if the point C (65, −28, −29) exists on the line.
c. The position vector e = −27î + 18ĵ + zk̂ terminates on the line formed between points A and B.
~
Calculate the value of z.
THINK WRITE
a. 1. Define the position vectors ~a and ~b. a. ⃗
OA = ~a = −3î + 6ĵ + 5k̂

OB = ~b = 5î + 2ĵ + 1k̂

68 Jacaranda Maths Quest 12 Specialist Mathematics Units 3 & 4 for Queensland


“c02VectorsInThreeDimensions_print” — 2019/8/7 — 6:51 — page 69 — #48

2. Calculate the direction vector, ~d, between ~d = ~b( − ~a ) ( )


points A and B. This vector is parallel to
~d = 5î + 2ĵ + 1k̂ − −3î + 6ĵ + 5k̂
the line.
~d = 8î − 4ĵ − 4k̂
3. Recall the formula for the vector equation ~r = ~a( + k ~d ) ( )
of a line and substitute in the expressions r = −3î + 6ĵ + 5k̂ + k 8î − 4ĵ − 4k̂
~
for ~d and ~a. The vector equation of the line
has been determined.
( ) ( )
b. 1. In most instances it is possible to verify a b. r
~ = −3î + 6ĵ + 5k̂ + k 8î − 4ĵ − 4k̂
solution by substituting values into an C (65, −28, −29)
equation. As k is unknown this is not Equate the î components to calculate k.
possible. The first(step is)to equate one of 65 = −3 + 8k
the components ~r î, ĵ, k̂ with the point
C (65, −28, −29).
2. Solve for k. 65 = −3 + 8k
65 + 3
=k
8
k = 8.5
( ) ( )
3. Substitute k = 8.5 into the vector equation r (8.5) = −3î + 6ĵ + 5k̂ + 8.5 8î − 4ĵ − 4k̂
~ ( ) ( )
~ to determine if the point C(65, −28, −29)
r = −3î + 6ĵ + 5k̂ + 68î − 34ĵ − 34k̂
lies on the line. = 65î − 28ĵ − 29k̂
∴ C(65, −28, −29) does lie on the line.
( ) ( )
c. 1. The vector equation of the line was c. r = −3î + 6ĵ + 5k̂ + k 8î − 4ĵ − 4k̂
~
developed in part a.
~e = −27î + 18ĵ + zk̂
Determine the value of k by equating Equate the î components to calculate k.
either the î or ĵ components from the −27 = −3 + 8k
vectors ~e and ~r. −27 + 3
=k
−8
k = −3
2. Substitute
( k = −3 into
) the( vector equation
) Let k = −3 ( ) ( )
r
~ = −3î + 6ĵ + 5k̂ + k 8î − 4ĵ − 4k̂ to ~r = −3î + 6ĵ + 5k̂ + −3 8î − 4ĵ − 4k̂
determine the position vector ~e. = −27î + 18ĵ + 17k̂
Hence the position vector ~e is
~e = −27î + 18ĵ + 17k̂
3. Equate the vectors ~r and ~e. −27î + 18ĵ + 17k̂ = −27î + 18ĵ + zk̂
4. Solve for z by equating the k̂ components. zk̂ = 17k̂
z = 17

2.5.4 Vector equations in parametric and Cartesian form


We have seen how lines can be expressed as vector equations in the form ~r = ~a + k ~d. These equations may
also be expressed in either parametric or Cartesian form. To demonstrate this, the vector equation from
Worked example 22 will be used.

CHAPTER 2 Vectors in three dimensions 69


“c02VectorsInThreeDimensions_print” — 2019/8/5 — 19:21 — page 70 — #49

Parametric form ( ) ( )
• The vector equation was ~r = −3î+6ĵ+5k̂ + k 8î−4ĵ−4k̂
• Points on the line are in the form P (x, y, z)
• Therefore, any point on the line is given by, (x, y, z) = (−3+8k, 6−4k, 5−4k)
The vector equation can be rewritten in parametric form by equating the components:

x = −3+8k, y = 6−4k, z = 5−4k

The parametric equation of a line in three-dimensional space is given by the general


formula
x = a1 + d1 k, y = a2 + d2 k, z = a3 + d3 k

Cartesian form
The position of the point P (x, y, z) is defined by the parameter k. Consequently, each of the equations that
exist in the parametric form of a line, can be rewritten in terms of k.
x = −3 + 8k y = 6 − 4k z = 5 − 4k

x+3 y−6 z−5


= k = k = k
8 −4 −4
x+3 y−6 z−5
k = k = k =
8 −4 −4

The line connecting points A and B can now be written in Cartesian form as the value for k is equivalent in
the three equations above. The Cartesian equation is given by,

x+3 y−6 z−5


= = =k
8 −4 −4
x+3 y−6 z−5
= =
8 −4 −4

The Cartesian equation of a line is likewise referred to as the symmetric equation of a line.

The Cartesian equation of line in three-dimensional space is given by the general


formula
x − a1 y − a2 z − a3
= =
d1 d2 d3

In this form, the line passes through the point (x, y, z) = (a1 , a2 , a3 ), and is parallel to the vector
d = d1 î + d2 ĵ + d3 k̂.
˜
The relationship
( between the
) three( forms of a line
) in vector space is summarised below using the vector
equation r = −3î + 6ĵ + 5k̂ + k 8î − 4ĵ − 4k̂ as an example.
˜

70 Jacaranda Maths Quest 12 Specialist Mathematics Units 3 & 4 for Queensland


“c02VectorsInThreeDimensions_print” — 2019/8/7 — 6:51 — page 71 — #50

Vector equation Parametric form Cartesian equation

( ) x+3 y−6 z−5


~r = −3î
(
+ 6ĵ + 5k̂ +
)
x = −3 + 8k
8
=
−4
=
−4
k 8î − 4ĵ − 4k̂ y = 6 − 4k
z = 5 − 4k
~a = −3î + 6ĵ + 5k̂ (a1 , a2 , a3 ) = (−3, 6, 5)
(a1 , a2 , a3 ) = (−3, 6, 5)
~d = 8î − 4ĵ − 4k̂
(d1 , d2 , d3 ) = (8, −4, −4) (d1 , d2 , d3 ) = (8, −4, −4)

All forms are appropriate when an equation is required to model a line in three-dimensional vector
geometry. The Cartesian equation is often preferred if you are required to calculate the point on the line that
intersects a plane. The intersection points occur when either x, y or z equal zero. Upon making the relevant
substitution, the location of the point P (x, y, z) can then be calculated.

WORKED EXAMPLE 23
( ) ( )
The vector equation of a line is given by ~r = 8î + 2ĵ − 3k̂ + k 3î − 4ĵ + 2k̂ .
a. Convert the vector equation into parametric form.
b. Express the parametric form of the line as a Cartesian equation.

THINK WRITE
a. 1. Write an equation linking the general location of the a. (x, y, z) = (8 + 3k, 2 − 4k, −3 + 2k )
point P (x, y, z) to the components of the vector
equation.
2. Equate the components to write three equations x = 8 + 3k
representing each of the three dimensions. y = 2 − 4k
z = −3 + 2k
b. 1. Rewrite the equation for the x-component in b. x = 8 + 3k
terms of k. x−8
k=
3
2. Rewrite the equation for the y-component in y = 2 − 4k
terms of k. y−2
k=
−4
3. Rewrite the equation for the z-component in z = −3 + 2k
terms of k. z+3
k=
2
x−8 y−2 z+3
4. Equate the three equations as they are equivalent = = =k
values of k. 3 −4 2
x−8 y−2 z+3
5. Express the final answer as a Cartesian equation in = =
terms of only x, y and z. 3 −4 2

CHAPTER 2 Vectors in three dimensions 71


“c02VectorsInThreeDimensions_print” — 2019/8/7 — 6:51 — page 72 — #51

Units 3 & 4 Area 2 Sequence 1 Concept 4


The vector equation of a straight line Summary screen and practice questions

Exercise 2.5 The vector equation of a straight line


Technology free
1. Calculate the two-dimensional vector equation for the line passing through each of the following pairs
of points (x, y).
a. A (2, 1) and B (6, 7) b. A (0, 6) and B (9, −12)
c. A (−4, 6) and B (−1, 12) d. A (−2, −4) and B (5, 3)
e. A (2, 8) and B (7, −1) f. A (−7, −5) and B (1, 15)
2. WE21 a. Determine a vector equation, in the two-dimensional space (x, y), for the line passing through

the points A (−2, 8) and B (5, −3).


b. Calculate the vector location of the point r on the line if the parameter k = 3.
~
3. Determine the location of the point P (x, y) on the following lines.
a. r = (4î + 2ĵ) + k (3î − 5ĵ) when k = 2 b. r = (−5î + 6ĵ) + k (−2î + ĵ) when k = −3
~ ~
4. Calculate the three-dimensional vector equation for the line passing through each of the following pairs
of points (x, y, z).
a. A (−2, 2, 1) and B (0, 6, 7) b. A (4, −1, 11) and B (−13, 9, −12)
c. A (5, 4, 11) and B (−1, 1, 8) d. A (6, −1, 2) and B (5, 3, −9)
e. A (−3, −2, 1) and B (5, 7, −13) f. A (1, 5, 8) and B (7, 0, −1)
5. WE22 a. Determine a vector equation, in the three-dimensional space (x, y, z), for the line passing

through the points A (−4, 7, 6) and B (6, 3, 2).


b. Verify if the point C (96, −33, −34) exists on the line.
c. The position vector e = −34î + 19ĵ + zk̂ terminates on the line formed between points A and B.
~
Calculate the value of z. ( )
6. Calculate the vector location of the point P î, ĵ, k̂ on the following lines.
( ) ( )
a. r = −3î + ĵ − 5k̂ + k 6î − 5ĵ − 2k̂ when k = 5
~ ( ) ( )
b. r = 4.5î + ĵ − 8k̂ + k 2î + ĵ + 3k̂ when k = −2
~
7. Verify, through suitable calculations, if the following points lie on each of the lines below.

Vector equation of the line Point (x, y, z)

a.
~r = (−3î + 2ĵ) + k (5î + 9ĵ) P (12, 29, 0)
( ) ( )
b. r = −7î + 4ĵ − 8k̂ + k î − 8ĵ + 3k̂ P (−3, −28, 2)
~
( ) ( )
c. r = 8î + 6ĵ − 5k̂ + k 2î − 2ĵ − 9k̂ P (4, 10, 13)
~
8. Determine the vector equation of the line through the point P (6, 5, 1) that is parallel to the vector
~u = 6î − 2ĵ + 3k̂.
9. Calculate the vector equation of a line that is parallel to the z-axis and through the point P (1, 2, 4).
( ) ( )
10. A line in 3D vector space is defined by the equation r = 4î + ĵ − 5k̂ + k −2î + 3ĵ + 5k̂ . The head
( ) ~
of the vector ⃗
OT = xî + 10ĵ + zk̂ touches the line. Calculate the values for x and z.

72 Jacaranda Maths Quest 12 Specialist Mathematics Units 3 & 4 for Queensland


“c02VectorsInThreeDimensions_print” — 2019/8/7 — 6:51 — page 73 — #52

11. WE23a The vector equation of a line is given by r = a + k d. Convert the following vector equations
~ ~ ~
into parametric
( form. ) ( ) ( )
a. r = −î + 3ĵ + 7k̂ + k 2î + 5ĵ − 3k̂ b. r = −7î + 8ĵ + 3k̂ + k (19î + 11ĵ + k)
~ ( ) ( ) ~
c. r = 5î + ĵ − 3k̂ + k 2î + 2ĵ − 6k̂
~
12. WE23b Express the following parametric equations of lines as Cartesian equations.
a. x = 1 + 5k, y = 2 + 5k, z = 3 + 5k b. x = −2 + 6k, y = 4 − 9k, z = 8 − 2k
c. x = 11 + 12k, y = 3 + 21k, z = 12 + 13k
13. Convert the following Cartesian equations into the form r = a + k d.
~ ~ ~
x−2 y−5 z+8 x+7 y+1 z−2
a. = = b. = =
3 2 5 −2 6 3
14. Determine the point at which the following lines intersect the specified plane.

Cartesian equation Plane

x−2 y−5 z+8


a. = = xy-plane
3 2 5

x+7 y+1 z−2


b. = = xz-plane
−2 6 3

Technology active
15. Calculate the distance between the point Q (−2, 1, 3) and the line defined by the equations,
x = 4 − t, y = t, z = 2 + 2t. Give your answer to two decimal places.
16. The position of an aircraft is defined by the vector
( ) ( )
equation ~r = 2î − k̂ + k î + ĵ + k̂ .
A second aircraft is flying in a straight-line path and its
location was recorded on two occasions, defined by the
vectors ~a = 5î + 8ĵ + 11k̂ and ~b = 9î + 17ĵ + 23k̂.
In the interest of public safety, determine if it is
possible for the two aircraft to cross paths.
Justify your solution with suitable calculations and
state the assumptions considered to be true when
developing the response.
17. Two lines are formed between the points D, E, F and G. The first line joins D (−1, 2, 1) and
E (3, −3, 5) . The second line passes through F (2, −1 , 4) and G (12, −2, 9). Calculate the acute angle
between the two lines.

2.6 The vector product


2.6.1 Multiplying vectors to produce vectors
Up until this point, the product of two vectors has only been calculated as a scalar quantity. The dot product
(or scalar or inner product) for two vectors ~a and ~b is defined by:
• ~a · ~b = |~a| |~b| cos 𝜃: when the magnitudes of, and angle between ~a and ~b are known.
• ~a · ~b = a1 b1 + a2 b2 + a3 b3 : when ~a and ~b are expressed in component form.
Applications of the dot product include:
• determining if vectors are perpendicular
• calculating the angle between two vectors
• calculating projections and components of vectors.

CHAPTER 2 Vectors in three dimensions 73


“c02VectorsInThreeDimensions_print” — 2019/8/7 — 6:51 — page 74 — #53

Is it possible to multiply two vectors so that their product isn’t restricted to a scalar quantity? The vector
product of two vectors ~a and ~b, unlike the scalar product, is a vector. For this reason, it is called the vector
product, and has applications in fields as diverse as geology, physics, mechanics and engineering.
Furthermore, it is extremely useful in describing relationships between lines and planes in three dimensions.
We have seen how two
a b
vectors, ~a and ~b, can be used to calculate ~ × ~ (the normal vector is out of the page)
and define the vector equation of a line
between two points. The same two vectors, b
~
~a and ~b, provided that they are not parallel,
θ
define a plane. A plane is a flat surface
a
extending indefinitely in all directions. ~

The orientation of the plane


can be described by a vector perpendicular
(orthogonal) to the plane defined by vectors
b a
~ × ~ (the normal vector is into the page)
~a and ~b. This perpendicular vector is called
a normal vector, (or simply the normal) and
is the cross product of the two vectors which
define the plane.

The vector product is defined as


~a × ~b = |~a| |~b| sin (𝜃) ~n̂
where 𝜃 is the angle between ~a and ~b,
~n̂ is the unit vector perpendicular to both ~a and ~b in the right-hand sense.

Note: Vector product is commonly called cross product.


The right-hand sense is applicable in a right-handed coordinate system, as previously described in
Section 2.2. The vector product produces a vector perpendicular to the vectors ~a and ~b, travelling in a
direction that is either out of the page or into the page. This is explained below.

Normal Vector ~n̂ 1 - out of the page


As the fingers on your right-hand curl from ~a and ~b, your thumb points in the direction of the normal vector
~n̂1 = ~a × ~b. In this case, it is pointing out of the page.
z

nˆ 1

a × b = nˆ 1 (out of the page)


~ ~

b b
~ ~
y
θ
a
~ a
~ a b
x ~×~

74 Jacaranda Maths Quest 12 Specialist Mathematics Units 3 & 4 for Queensland


“c02VectorsInThreeDimensions_print” — 2019/8/7 — 6:51 — page 75 — #54

Normal Vector ~n̂ 2 - into the page


In order to curl your right hand from ~b to ~a, you would have to rotate your wrist so your thumb pointed
down. The normal vector, ~n̂2 = ~b × ~a, in this instance points into the page.

b
~

θ b
a ~
~

a
~

b a nˆ (into the page) nˆ 2


~×~= 2

Remember the direction of the normal vector by thinking of it as turning on a water tap.
• Turning a tap in a clockwise direction moves it inward (into the page).
• Rotating the tap head anti-clockwise moves it in an upward direction (out of the page).

WORKED EXAMPLE 24
The vectors ~a, ~b and ~c exist in a horizontal plane. Calculate the following vector products and in
each case state the direction of n ~̂.
a. a × b b. b × c
~ ~ ~ ~

a
~
c
~
4
6
82° 60° b
~
7

THINK WRITE
a. 1. Recall the vector product formula. a.
~a × ~b = |~a||~b| sin 𝜃 ~n̂
2. Substitute the values for |a| = 4, |b| = 7 and 𝜃 = 60°. = 4 × 7 × sin(60°) ~n̂
~ ~ √
3
= 28 × n̂
2 ~

3. Calculate the vector product, including the direction. = 14 3~n̂ into the page
Recognise that ~a turns to ~b in a clockwise direction,
indicating the direction of ~n.̂
b. 1. Recall the vector product formula. b.
~b × ~c = |~b||~c| sin 𝜃 ~n̂
2. Substitute the values for |b| = 7, |c| = 6 and 𝜃 = 142°. = 7 × 6 × sin(142°) ~n̂
~ ~
3. Calculate the vector product. Recognise that b rotates
~ = 25.86~n̂ out of the page
anti-clockwise to ~c, indicating the direction of ~n.̂

CHAPTER 2 Vectors in three dimensions 75


“c02VectorsInThreeDimensions_print” — 2019/8/7 — 6:51 — page 76 — #55

WORKED EXAMPLE 25
[ √ ] [ ]
𝜋 5𝜋
The vectors ~c = 2 2, and ~d = 6, exist in a horizontal plane. Calculate the exact value
4 6
of the vector product, ~c × ~d.

THINK WRITE
[ ]
1. Recognise that the vectors are in polar form, ~v = |~v|, 𝜃 and The magnitudes of the
state their magnitude. vectors√ are,
|~c| = 2 2 and |~d| = 6.
2. Sketch the vectors to get a clearer perspective of their Both vectors are in a
orientation. positive direction from the
y
x-axis. The angle, 𝜃,
between the vectors is,
5𝜋 𝜋 7𝜋
𝜃= − =
6 4 12

d c
~ ~

6 π
4
x

3. Recall the rule for vector product, ~a × ~b = |~a| |~b| sin 𝜃 ~n.̂ The ~a × ~b = |~a||~b| sin 𝜃 ~n̂ ( )
direction of the normal vector can be determined by applying √ 7𝜋
c × d = 2 2 × 6 × sin n̂
the right hand rule. In this case, ~c × ~d is turning in an ~ ~ 12 ~
anticlockwise direction, which is out of the page. √ ( )
7𝜋
= 12 2 sin n̂
12 ~
out of the page.

2.6.2 Applications: the area of parallelograms and triangles


The vector product can be used to calculate the area of particular shapes. The area of a parallelogram is
given by the rule, Area = base × height. Consider the relationship between the area of a parallelogram and
the vector product.
The height of the parallelogram can be expressed using basic trigonometric ratios.

opposite
sin 𝜃 =
hypotenuse
h
sin 𝜃 =
|~b|
b h
h = |~b| sin 𝜃 ~

a
The area of the parallelogram is then given by. ~
𝜃
Area = base × height
= |~a| × |~b| sin 𝜃
= |~a||~b| sin 𝜃

76 Jacaranda Maths Quest 12 Specialist Mathematics Units 3 & 4 for Queensland


“c02VectorsInThreeDimensions_print” — 2019/8/7 — 6:51 — page 77 — #56

Recalling the formula for the vector product, ~a × ~b = |~a||~b| sin 𝜃 ~n,̂ we can recognise that the area of a
parallelogram, defined by the vectors ~a and ~b, is equal to the magnitude of the vector product of ~a and ~b.
That is, the area is given by:

Areaparallelogram = |~a × ~b|.

The vector product can be used to calculate the area of a triangle via a similar procedure.
1
Recall that the area of any triangle is given by, Area = × base × height.
2
Given that h = |~b| sin 𝜃, the area can be calculated

1
Area = × |a| × |~b| sin 𝜃
2 ~

Thus, the area of a triangle, defined by the vectors ~a and ~b, is equal to,

1 b
Areatriangle = |~a × ~b| ~ h
2
1
= |~a| × |~b| sin 𝜃
2 a
~
1
= |~a||~b| sin 𝜃
2

WORKED EXAMPLE 26
a. Use the vector formula, Area = |~a||~b|sin 𝜃 to determine the area of the parallelogram formed by
the vectors with adjacent sides, ~a and ~b. Give your answer to 2 decimal places.

│~a│= 5

𝜃 = 60°
│~b│= 4
1
b. Use the formula Area = |~a||~b|sin 𝜃 to calculate the area of the triangle. Give your answer to 2
2
decimal places.

│~a│= 2

86.5°
│~b│= 3.2

CHAPTER 2 Vectors in three dimensions 77


“c02VectorsInThreeDimensions_print” — 2019/8/7 — 6:51 — page 78 — #57

THINK WRITE
a. 1. Recall and state the formula for the area of a a. Area = |~a||~b| sin 𝜃
parallelogram.
Area = |~a||~b| sin 𝜃
2. Substitute the values for |a| = 5, |b| = 4 and = 5 × 4 × sin 60°
~ ~ √
𝜃 = 60°. 3
= 20 ×
2

3. Calculate the area and simplify the answer to two = 10 3
decimal places. ≈ 17.32 square units
1
b. 1. Recall and state the formula for the area of a b. Area = |a||b| sin 𝜃
1 2~ ~
triangle. Area = |~a||~b| sin 𝜃
2
1
2. Substitute the values for |~a| = 2, |~b| = 3.2 and = × 2 × 3.2 × sin 86.5°
𝜃 = 86.5°. 2
3. Calculate the area and simplify the solution to two ≈ 3.19 square units
decimal places.

2.6.3 The vector product in î − ĵ − k̂ form


Recall that the vectors î, ĵ and k̂ are the mutually perpendicular unit
vectors that can be used to describe three-dimensional space. The
x-, y- and z- directions cannot be arranged in any order, even though
they may be mutually perpendicular. The form is a right-hand kˆ
triad. This means that the direction of z must be that obtained
by anticlockwise rotation of x to y.
Consider some special vector products. Remember that
~ ~b = |~a||~b| sin 𝜃 ~n̂ where ~n̂ is a unit vector perpendicular to ~a and ~b
a × î ĵ
in the right-hand sense.
î×î ĵ × k̂ î × k̂

~a = î and ~b = î ~a = ĵ and ~b = k̂ ~a = î and ~b = k̂

~a × ~b = |~a||~b| sin 𝜃 ~n̂ ĵ × k̂ = |~a||~b| sin 𝜃 ~n̂ ~a × ~b = |~a||~b| sin 𝜃 ~n̂


î × î = 1 × 1 sin (0°) ~n̂ ĵ × k̂ = 1 × 1 sin (90°) ~n̂ î × k̂ = 1 × 1 sin (90°) ~n̂
ĵ × k̂ = 1~n̂ in the direction of + î î × k̂ = 1~n̂ in the direction of − ĵ
î × î = ~0

∴ î × î = ~0 ∴ ĵ × k̂ = î ∴ î × k̂ = −ĵ

Continuing in this way, we can show that the cross product for the basic unit vectors î, ĵ and k̂ are:

î × î = ~0 î × ĵ = k̂ î × k̂ = −ĵ
ĵ × î = −k̂ ĵ × ĵ = ~0 ĵ × k̂ = î
k̂ × î = ĵ k̂ × ĵ = − î k̂ × k̂ = ~0

78 Jacaranda Maths Quest 12 Specialist Mathematics Units 3 & 4 for Queensland


“c02VectorsInThreeDimensions_print” — 2019/8/7 — 6:51 — page 79 — #58

A useful way of summarising these results is shown in the figure.


A product following the direction indicated (clockwise) produces ĵ
a positive value.

For example, î × ĵ = k̂ î

A product in the anticlockwise direction produces a negative value.


For example, k̂ × ĵ = −î

To evaluate the vector product of two vectors that are expressed in component form we can use the
Distributive Law. This is illustrated in the following worked example.

WORKED EXAMPLE 27

If a = 3î + 2ĵ − 4k̂, b = −2î + 3ĵ − k̂ and ~c = −3î + 4ĵ − 7k̂, calculate the following vector products
~ ~
using the distributive law of vectors.
a. a × b
~ ~
b. c × b
~ ~
c. Justify that the vector product from b is orthogonal to vectors c and b respectively.
~ ~

THINK WRITE
( ) ( )
a. 1. Express ~a and ~b as a product in component a. a ×
~ ~ b = 3î + 2ĵ − 4k̂ × −2î + 3ĵ − k̂
form.
2. Calculate the product a × b by applying = 3î × −2î + 3î × 3ĵ + 3î × −k̂
~ ~
the Distributive Law for vectors. +2ĵ × −2î + 2ĵ × 3ĵ + 2ĵ × −k̂
+ − 4k̂ × −2î + −4k̂ × 3ĵ + −4k̂ × −k̂
3. Simplify the calculation by collecting like = 9k̂ + 3ĵ + 4k̂ − 2î + 8ĵ + 12î
vector components. = 10î + 11ĵ + 13k̂
( ) ( )
b. 1. Express c and b as a product in component b. c × b = −3î + 4ĵ − 7k̂ × −2î + 3ĵ − k̂
~ ~ ~ ~
form.
2. Calculate the product a × b applying the = −3î × −2î + −3î × 3ĵ + −3î × −k̂
~ ~
Distributive Law for vectors. +4ĵ × −2î + 4ĵ × 3ĵ + 4ĵ × −k̂
+ − 7k̂ × −2î + −7k̂ × 3ĵ + −7k̂ × −k̂
3. Simplify the calculation by collecting like = −9k̂ − 3ĵ + 8k̂ − 4î + 14ĵ + 21î
vector components. = 17î + 11ĵ − k̂
c. 1. Let c × b = n. Vectors, c and n, are c. Let c × b = n
~ ~ ~ ~ ~ ~ ~ ~
perpendicular if ~c · ~n = 0. c and ~n(are orthogonal)if ~c( · ~n = 0
~ )
2. Calculate c · n by recalling the formula c · n = −3î + 4ĵ − 7k̂ · 17î + 11ĵ − k̂
~ ~ ~ ~
~c · ~n = x1 x2 + y1 y2 + z1 z2 . Multiply the = −51 + 44 + 7
corresponding components. =0
3. Vectors, b and n, are perpendicular if
~ ~b and ~n(are orthogonal if b · n = 0
~ ) (~ ~ )
b · n = 0. Calculate ~c · ~n by recalling the
~ ~ ~b · ~n = −2î + 3ĵ − k̂ · 17î + 11ĵ − k̂
formula ~c · ~n = x1 x2 + y1 y2 + z1 z2 . = −34 + 33 + 1
Multiply the corresponding components. =0

CHAPTER 2 Vectors in three dimensions 79


“c02VectorsInThreeDimensions_print” — 2019/8/7 — 6:51 — page 80 — #59

4. Write a justifying statement. ∴ The vector product ~c × ~b = ~n is orthogonal to


both vectors ~c and ~b.

TI| THINK WRITE CASIO| THINK WRITE


a.1. On a Calculator page, a.1. On a Run-Matrix screen,
select select MAT/VCT
MENU Define Vectors
7: Matrix Vector A and B as shown in
C: Vector Worked Example 1b.
2: Cross Product
Press the ENTER
button.

2. Complete
[ the entry
] line 2. On a Run-Matrix screen,
([ −3, 2, −4] , select OPTN MAT/VCT
−2, 3, −1 ) ⊳ (x2 times)CrossP(

3. Press the ENTER 3. Complete the entry


button. The answer line as
appears on the screen. Vct ALPHA
X, 𝜃, T, Vct ALPHA)
Log)

4. Press the EXE button.


The answer appears on
the screen

2.6.4 Calculating the vector product using determinants


The Distributive Law for vectors is a valid method for calculating the vector product when the vectors are in
component form. The process is rather arduous, and the same result can be calculated more efficiently using
determinants. First, we shall review the definitions of a determinant in both two and three dimensions.
For a 2 × 2 matrix:
[ ]
a b |a b|
If A = then, det (A) = | | = ad − bc
c d | c d|

For a 3 × 3 matrix:

⎡a b c⎤ |a b c|
| e f| |d f| |d e|
If A = ⎢ d e f ⎥ then, det (A) = ||d e f || = a | | − b| | + c| |
⎢ ⎥ |h i| |g i| |g h|
⎣g h i ⎦ |g h i |

80 Jacaranda Maths Quest 12 Specialist Mathematics Units 3 & 4 for Queensland


“c02VectorsInThreeDimensions_print” — 2019/8/7 — 6:51 — page 81 — #60

The elements of a determinant are generally numerical figures. It is possible to | î ĵ k̂|


replace the first row of the matrix with the vectors î, ĵ and k̂. When calculating the ||d e f ||
determinant in this case, the sub-determinants (minors) become scalar |g h i |
multiples of the vectors î, ĵ and k̂.

| î ĵ k̂ |
|d e f | = î ||e f || − ĵ ||d f || + k̂ ||d e||
| | |h i | |g i | |g h|
|g h i |
( ) ( )
If the vectors exist in the form, ~a = a1 î + a2 ĵ + a3 k̂ and ~b = b1 î + b2 ĵ + b3 k̂ , then the vector
product can be defined in terms of determinants as,

|î ĵ k̂ |
| | |a2 a3 | |a1 a3 | |a1 a2 |
~a × ~b = ||a1 a2 a3 || = î ||b b || − ĵ ||b b || + k̂ ||b b ||
2 3 1 3 1 2
|b1 b2 b3 |
= î (a2 b3 − a3 b2 ) − ĵ (a1 b3 − a3 b1 ) + k̂(a1 b2 − a2 b1 )

Hopefully it is now clearer as to why the vector product can also be called the cross product.

WORKED EXAMPLE 28

a. Calculate ~a × ~b using determinants, given ~a = −î + 4ĵ + 6k̂ and ~b = 4î + 2ĵ − 5k̂.


b. Calculate ~b × ~a to investigate if the vector product is commutative.

THINK WRITE
| î ĵ k̂ |
a. 1. Express the vector product, ~a × ~b |
a. a × b = | −1 4 6 ||
~ ~
as the determinant of a 3 × 3 | 4 2 −5 |
matrix.
| 4 6| | −1 6 | | −1 4 |
2. Calculate the determinant for the = î| | − ĵ| | + k̂ | |
| 2 −5 | | 4 −5 | | 4 2|
matrix.
= î (4 × −5 − 6 × 2) − ĵ (−1 × −5 − 6 × 4)
Remember to write out the
expansion with the signed minors +k̂(−1 × 2 − 4 × 4)
(cofactors). = î(−20 − 12) − ĵ(5 − 24) + k̂(−2 − 16)
3. Complete the expansion being = −32î + 19ĵ − 18k̂
careful with + and − signs.
| î ĵ k̂ |
b. 1. Express the vector product, ~b × ~a b. b × a = | 4 2 −5 |
~ ~ | |
as the determinant of a 3 × 3 | −1 4 6|
matrix.
| 2 −5 | | 4 −5 | | 4 2|
2. Calculate the determinant for the = î| | − ĵ| | + k̂ | |
| 4 6| | −1 6 | | −1 4 |
matrix. Remember to write out the
= î (2 × 6 − −5 × 4) − ĵ (4 × 6 − −5 × −1)
expansion with the signed minors
(cofactors). +k̂(4 × 4 − 2 × −1)
= î (12 + 20) − ĵ (24 − 5) + k̂(16 + 2)
3. Complete the expansion being = 32î − 19ĵ + 18k̂
careful with + and − signs.

CHAPTER 2 Vectors in three dimensions 81


“c02VectorsInThreeDimensions_print” — 2019/8/7 — 6:51 — page 82 — #61

4. Recall that an operation is The cross product is not commutative because


commutative if a ◦ b = b ◦ a. ~ × ~b ≠ ~b × ~a
a
Conclude with a statement in
response to the investigation.

2.6.5 Properties of the vector product


Although the vector product does not form a commutative operation with vectors, it is interesting to note
that ~a × ~b = −~b × ~a. This property is known as being anti-commutative. The following six theorems
summarise some of the properties of the vector product.

Six properties of the cross product


1. ~a × ~(b = −~)b × ~a
2. a( × ~b )+ ~c = ~a × ~b + ~a × ~c
3. ~a +( ~b ×)~c =(~a × ~c)+ ~b × ~c
4. ~a · (~b × ~c )= (~a × ~b ) · ~c ( )
5. ~a × ~b × ~c = ( ~ ~a · c
) ~b − ×
~ ~ ~c
a b
6. (k~a ) × ~b = k ~a × ~b = ~a × (k~b)

where ~a, ~b and ~c are vectors, and k is a scalar.

2.6.6 The scalar triple product


( )
The fourth theorem, ~a · ~b × ~c , is referred to as the scalar triple product. As the name suggests, the
product is a scalar quantity, and is sometimes referred to as the triple product.
The triple product has numerous applications, one of which is
related to the volume of a parallelepiped. The volume of a a
~
parallelepiped whose adjacent sides are the vectors ~a, ~b and ~c is,
( ) θ
c
Volume = ||~a · ~b × ~c ||
~
b
~
In three steps, this formula can be proved by combining the theories of both the dot and vector products.
1. The area of the base is given by the vector product A = |~b × ~c|.
2. If 𝜃 is the angle between ~a and ~b × ~c, the height of the parallelepiped will be h = |~a|| cos 𝜃|.
3. The vector formula for the volume of a parallelepiped is then developed by combining the equations,
A = |~b × ~c| and h = |~a|| cos 𝜃|,

Volume = A × h
= |~a|| cos 𝜃| × |~b × ~c|
( )
= |~a · ~b × ~c |
( )
Determinants can be used to simplify the triple product calculation, ~a · ~b × ~c .
( ) ( ) ( )
If ~a = a1 î + a2 ĵ + a3 k̂ , ~b = b1 î + b2 ĵ + b3 k̂ and ~c = c1 î + c2 ĵ + c3 k̂ , the scalar triple product can
be calculated by

82 Jacaranda Maths Quest 12 Specialist Mathematics Units 3 & 4 for Queensland


“c02VectorsInThreeDimensions_print” — 2019/8/7 — 6:51 — page 83 — #62

|a1 a2 a3 |
( ) | |
~a · ~b × ~c = ||b1 b2 b3 |
|
| c1 c2 c3 |

WORKED EXAMPLE 29
Determine the volume of the parallelepiped containing the adjacent vectors, ~a, ~b and ~c.

~a = î + 2ĵ + k̂, ~b = 5î + 2ĵ − 4k̂ and ~c = 3î − 4ĵ − k̂.

THINK WRITE
( )
1. State the formula for the volume, which is Volume = |~a · ~b × ~c |
the scalar triple product.
|1 2 1|
2. Express the scalar triple product as the |
V = |5 2 −4 ||
determinate of a 3 × 3 matrix. Calculate | 3 −4 −1 |
the determinant of the matrix.
| 2 −4 | | 5 −4 | |5 2|
3. Remember to write out the expansion with V = 1| | − 2| | + 1| |
| −4 −1 | | 3 −1 | | 3 −4 |
the signed minors (cofactors).
4. Complete the expansion being careful with V = | − 18 − 14 − 26|
+ and − signs. = | − 58|
= 58

TI| THINK WRITE CASIO| THINK WRITE


1. On a Calculator page, 1. On a Run-Matrix screen,
select MENU select
7: Matrix & Vector MAT/VCT
1: Create To switch between the
1: Matrix vector and matrix screens,
Press the ENTER press the M ⇔ V button.
button.

2. Complete the entry line 2. Press the EXE button.


as 3 3 Press the OK Complete the entry line as
button. 3 3 Press the EXE button.

3. Complete the entry 3. Complete the entry


line as line as
1 2 1 1 2 1
5 2 −4 5 2 −4
3 −4 −1 3 −4 −1
Press the ENTER Press the EXIT button when
button when finished. finished.

CHAPTER 2 Vectors in three dimensions 83


“c02VectorsInThreeDimensions_print” — 2019/8/7 — 6:51 — page 84 — #63

4. On a Calculator 4. On a Run-Matrix screen,


page, select MENU select OPTN
7: Matrix & Vector MAT/VCT
3: Determinant DET
CTRL (-)

5. Press the ENTER 5. Complete the entry line as


button. SHIFT
The answer appears on 2
the screen. ALPHA
X, 𝜃, T
Note: The absolute
value of determinant is
the volume of the
parallelepiped.

6. Press the EXE button.


The answer appears on the
screen.
Note: The absolute value of
determinant is the volume of
the parallelepiped.

Units 3 & 4 Area 2 Sequence 1 Concept 5


The vector product Summary screen and practice questions

Exercise 2.6 The vector product


Technology active
1. WE24 The vectors ~a and ~b exist in a horizontal plane. Calculate ~a × ~b, and state the direction of ~n.̂
a. b b. a
~ ~

5
6.5
82°
110°
8 3
a
~
b
~
c. b d. 180°
~ a
15 ~
b
~ 4.5
30° 2.1

7
a
~
2. WE25 The vectors a and b exist in a horizontal plane. Calculate the vector product, a × b, for the
~ ~ ~ ~
following pairs of vectors.
a. a = [12, 25°] and b = [10, 75°] b. a = [21, 182°] and b = [16, 267°]
~ ~ ~ ~
c. a = [4, 173°] and b = [7, 22°] d. a = [5, 25°] and b = [7, 345°]
~ ~ ~ ~

84 Jacaranda Maths Quest 12 Specialist Mathematics Units 3 & 4 for Queensland


“c02VectorsInThreeDimensions_print” — 2019/8/7 — 6:51 — page 85 — #64

1
3. WE26b Use the formula Area = |~a||~b| sin 𝜃 to calculate the area of the following triangles.
2
a. b.
138°
│~
a│= 4.5
│~
b│= 8
a│= 3.1
│~

53°
│~
b│= 6
4. WE26a Use the vector formula, Area = |a||b| sin 𝜃 to determine the area of the parallelograms formed
~ ~
by the vectors with adjacent sides, ~a and ~b.
a.
│~
a│= 4

30°
│~
b│= 2.6
b.

│~
a│= 5.5
145°

b│= 9
│~
5. The table below summaries the nine possibilities when the vector product is calculated using the unit
vectors î, ĵ or k̂. Complete the table by using your knowledge of the Distributive Law for vectors.

Second vector b
~
× î ĵ k̂

î −ĵ

First Vector a ĵ ~0
~

6. Use the vector product of vectors to calculate a vector perpendicular to both ~a and ~b.
a.
~a = 2î + 3ĵ − 7k̂ and ~b = −î + 7ĵ − 2k̂ b.
~a = −5î − ĵ − 3k̂ and ~b = 11î − 4ĵ − 2k̂
c. a = î + 12ĵ + 8k̂ and b = −9î + 3ĵ + 7k̂ d. a = 3.5î + 1.5ĵ − 2k̂ and b = 0.5î − 2ĵ
~ ~ ~ ~
7. WE27 If a = 4î + 3ĵ − 2k̂, b = −3î + 5ĵ − k̂ and c = −7î + ĵ − 2k̂, calculate the following vector
~ ~ ~
products using the Distributive Law for vectors
a. a × b b. c × b
~ ~ ~ ~
c. Justify that the vector product from b, is orthogonal to vectors c and b respectively.
~ ~
8. The two vectors ~a = 3î − ĵ + 2k̂ and ~b = aî − 2ĵ + 4k̂ exist in the same three-dimensional space, and
~a · ~b = 28. Calculate ~a × ~b to determine if it is possible to find a unit vector orthogonal to ~a and ~b.
9. WE28 Calculate ~a × ~b using determinants, given ~a = −3î + 7ĵ − 5k̂, ~b = 8î + ĵ − 2k̂.
a.
b. Calculate b × a to investigate if the vector product is commutative.
~ ~
10. Use determinants to calculate the following vector products, a × b.
~ ~
a.
~a = 5î − 2ĵ − 9k̂ and ~b = 2î + 3ĵ + 4k̂ b.
~a = 3ĵ − 8k̂ and ~b = −4î + 6ĵ + 7k̂
c. a = 3î − 2ĵ − 4k̂ and b = 7î + 2ĵ − k̂ d. a = −7î + ĵ − 6k̂ and b = 6î − 12ĵ + 13k̂
~ ~ ~ ~

CHAPTER 2 Vectors in three dimensions 85


“c02VectorsInThreeDimensions_print” — 2019/8/7 — 6:51 — page 86 — #65

11. WE29 Determine the volume of the parallelepiped containing the adjacent vectors, ~a, ~b and ~c.
a.
~a = 5î + 3ĵ + 2k̂, ~b = 7î + 3ĵ − 2k̂ and ~c = 8î − 7ĵ − 3k̂
b. a = î − 2ĵ + 8k̂, b = −3î − 2ĵ + 9k̂ and c = −5î + 6ĵ − 2k̂
~ ~ ~
c.
~a = 8î + 3ĵ − 7k̂, ~b = −4î + 3ĵ + k̂ and ~c = 7î − 11ĵ + 6k̂
d. a = 7î + 5ĵ + 6k̂, b = −2î − 4ĵ − 6k̂ and c = 2î − 4ĵ + 8k̂
~ ~ ~
12. The vertices of a triangle are defined by the vectors, ~a = 7î + 4ĵ + 8k̂, ~b = î + 14ĵ − 8k̂ and
1
~c = 8î + 3ĵ + 2k̂. Calculate the area using the rule A = 2 |~a × ~b|.
13. Use the vector product to prove the property ~a × ~a = ~0, given that ~a = (x, y, z).
14. Calculate the scalar triple product for the vectors, ~a = 3î + 3ĵ − 6k̂, ~b = 2î + 4ĵ and ~c = 3î + 9ĵ + 6k̂, to
determine if they are coplanar (exist on the same plane).
15. The vector product of two vectors ~a = 3î + 5ĵ − 3k̂ and ~b = 4î + by ĵ + bz k̂ is ~a × ~b = −7î − 15ĵ − 32k̂.
Calculate the values of by and bz .
16. Calculate each of the following scalar triple products.
a. ĵ · k̂ × k̂ b. î · ĵ × k̂ c. î · k̂ × ĵ d. k̂ · ĵ × î
17. Use the vector product to calculate the angle between the vectors ~a = 3î + 4ĵ − 2k̂ and ~b = 2î + ĵ + 2k̂.
18. A tetrahedron is a pyramid consisting of four triangular faces. The volume of a tetrahedron is given by
1 ( )
the rule, V = ||~a · ~b × ~c ||. Calculate the volume of a tetrahedron with vertices
6
A (6, 2, 0), B (7, 5, 0), C (3, 8, 1) and D (13, 7, 2).
19. Verify through suitable calculations, that the quadrilateral with vertices defined by ~a, ~b, ~c and ~d is a
parallelogram. Hence, calculate its area using determinants.
~a = 6î + 3ĵ + k̂
~b = 3î + 7ĵ + 4k̂
~c = 3î + 5ĵ + 9k̂
~d = 6î + ĵ + 6k̂

2.7 Applications of vectors


2.7.1 The vector and Cartesian equation of a plane in three dimensions
A two-dimensional line can be represented by the scalar equation y = mx + c. In three-dimensional space,
the line between two points is modelled by the vector equation of a line, ~r = ~a + k ~d. In this section we will
model a plane in three dimensions through the application of vectors. A plane can be described as a ‘flat’
2-dimensional surface extending indefinitely. To help you visualise a plane, three graphical examples are
shown below.
z
z z

x y

x y x

86 Jacaranda Maths Quest 12 Specialist Mathematics Units 3 & 4 for Queensland


“c02VectorsInThreeDimensions_print” — 2019/8/7 — 6:51 — page 87 — #66

Determining the equation of a plane using a point and the normal


The equation of a plane written in Cartesian form, z
is expressed as ax + by + cz + d = 0. To calculate the
equation of a plane, two important features are required:
1. A point P (x, y, z) that exists on the plane. ~n
2. A vector that is perpendicular to the plane.
Remember this vector is called the normal.
Consider the diagram shown. The locations of points P0
r a
P and P0 can be defined by position vectors. ~ –~
That is, P (x, y, z) = ~r and P0 (a, b, c) = ~a. The P
vector connecting the two points can then a
~
be defined as ⃗ P0 P = ~r − ~a . r
~
As we are asserting that ~r − ~a lies on the plane it
will always be perpendicular to the normal. That is,
where P0 = (a, b, c) and P = (x, y, z)
( ) x y
~r − ~a · ~n = 0
This is defined as the vector equation of a plane.

The vector equation of a plane is given by


( )
~r − ~a · ~n = 0
This can be written as ~r · ~n = ~a · ~n

To translate this into Cartesian form, the vector, ~r − ~a is to be expressed in terms of its Cartesian
coordinates.


P0 P = ~r − ~a
( ) ( )
~ − ~a = xî + yĵ + zk̂ − aî + bĵ + ck̂
r
= (x − a) î + (y − b) ĵ + (z − c) k̂

In component form, the normal vector is ~n = n1 î + n2 ĵ + n3 k̂. The dot product calculation results in:
( )
~r − ~a · ~n = 0
[ ] [ ]
(x − a) î + (y − b) ĵ + (z − c) k̂ · n1 î + n2 ĵ + n3 k̂ = 0
n1 (x − a) + n2 (y − b) + n3 (z − c) = 0

This is the equation of a plane containing the point, P0 (a, b, c), that is perpendicular to
~ n1 î + n2 ĵ + n3 k̂.
n =
The equation can then be transformed into Cartesian form, ax + by + cz + d = 0, by expanding out and
collecting like terms. It is interesting to note that the coefficients in this form are the components of the
normal vector.

The equation of a plane containing the points P (x, y, z) and P0 (a, b, c) that is
n = n1 î + n2 ĵ + n3 k̂ is given by
perpendicular to the normal ~
n1 (x−a) + n2 (y−b) + n3 (z−c) = 0

CHAPTER 2 Vectors in three dimensions 87


“c02VectorsInThreeDimensions_print” — 2019/8/7 — 6:51 — page 88 — #67

WORKED EXAMPLE 30

n = 2î + 3ĵ + 4k̂, containing the point


Calculate the equation of the plane perpendicular to ~
P0 (2, 4, −1) using a dot product calculation.

THINK WRITE
1. Define the vectors ~r, ~a and ~n. ~r = (x, y, z), ~a = (2, 4, −1) and
n = 2î + 3ĵ + 4k̂
(~ )
2. Determine the vector ~r − ~a by applying the ~r − ~a = (x − 2) î + (y − 4) ĵ + (z + 1) k̂
formula
⃗P0 P = ~r − ~a
= (x − a) î + (y − b) ĵ + (z − c) k̂
( )
3. Recall the formal dot product definition for ~r − ~a · ~n = 0
( ) ( )
a plane and substitute in the vectors. (x − 2) î + (y − 4) ĵ + (z + 1) k̂ · 2î + 3ĵ + 4k̂ = 0
4. Complete the dot product calculations. 2 (x − 2) + 3 (y − 4) + 4 (z + 1) = 0
5. Expand and simplify. 2x − 4 + 3y − 12 + 4z + 4 = 0
2x + 3y + 4z − 12 = 0

The procedure completed in Worked example 30 was very detailed. The same result can be achieved in
fewer steps by recalling the formula, n1 (x − a) + n2 (y − b) + n3 (z − c) = 0. This is demonstrated in the
next example and is the preferred option unless specified otherwise.

WORKED EXAMPLE 31

n = 2î + 4ĵ − 7k̂ is
Determine the equation of the plane containing the point (−5, 3, 6) if the vector ~
normal to it.

THINK WRITE
1. Recall the general formula for the equation of a n1 (x − a) + n2 (y − b) + n3 (z − c) = 0
plane.
2. Substitute the components of point (a, b, c) and n.
~ 2 (x + 5) + 4 (y − 3) − 7 (z − 6) = 0
Expand and collect like terms. 2x + 10 + 4y − 12 − 7z + 42 = 0
2x + 4y − 7z + 40 = 0
3. State your answer. The equation of the plane is
2x + 4y − 7z + 40 = 0

88 Jacaranda Maths Quest 12 Specialist Mathematics Units 3 & 4 for Queensland


“c02VectorsInThreeDimensions_print” — 2019/8/7 — 6:51 — page 89 — #68

z
Determining the equation of a plane using three points
In both worked examples above, the equation was calculated P (x1, y1, z1)
based on a known point on the plane and a vector normal to it.
Consider the plane shown, defined by the three non-collinear points. Q (x2, y2, z2)
A normal vector can be calculated by applying our knowledge of the
vector product. The following example demonstrates this procedure.

x R (x3, y3, z3)

WORKED EXAMPLE 32
a. Calculate the Cartesian equation of the plane determined by the points P (−2, 1, 3), Q (3, 5, 6)
and R (−5, 3, 5)
b. Determine the axis intercepts for the plane.

THINK WRITE
a. 1. Calculate and define two vectors relating a. ⃗
PQ = (3 − −2) î + (5 − 1) ĵ + (6 − 3) k̂
to the points P, Q and R. = 5î + 4ĵ + 3k̂
⃗PR = (−5 − −2) î + (3 − 1) ĵ + (5 − 3) k̂
= −3î + 2ĵ + 2k̂
2. Find a vector perpendicular to ⃗ PQ and ⃗ ⃗
~n = PQ × PR
⃗PR . This can | î ĵ k̂ |
= | |
be found by calculating ⃗ PQ × ⃗ PR . This is n
~ | 5 4 3|
calculated as the determinant. | −3 2 2 |
3. State the normal vector. ~n = 2î − 19ĵ + 22k̂
4. Recall the general formula for the equation n1 (x − a) + n2 (y − b) + n3 (z − c) = 0
of a plane.
5. Substitute the components of point 2(x + 2) − 19(y − 1) + 22(z − 3) = 0
P (−2, 1, 3) and the normal vector into the 2x + 4 − 19y + 19 + 22z − 66 = 0
formula. 2x − 19y + 22z − 43 = 0
6. State your answer. ∴ 2x − 19y + 22z − 43 = 0
b. 1. Calculate the x-intercept (x, 0, 0) when b. Let y = 0 and z = 0,
y = 0, z = 0. 2x − 19 (0) + 22 (0) = 43
43
x=
2
2. Calculate the y-intercept (0, y, 0) when Let x = 0 and z = 0,
x = 0, z = 0. 2 (0) − 19y + 22 (0) = 43
43
y=−
19
3. Calculate the z-intercept (0, 0, z) when Let x = 0 and y = 0,
x = 0, y = 0. 2 (0) − 19 (0) + 22z = 43
43
z=
22
4. State the three intercepts. The
( plane)intersects
( the )
axes at( )
43 43 43
, 0, 0 , 0, − , 0 and 0, 0,
2 19 22

CHAPTER 2 Vectors in three dimensions 89


“c02VectorsInThreeDimensions_print” — 2019/8/7 — 6:51 — page 90 — #69

If the equation of a plane is written in the form ax + by + cz = d, the intercepts can be calculated by the
d d d
following formulae, x = , y = , z = .
a b c

2.7.2 Torque
The vector product has further applications in both engineering and physics. Consider tightening a bolt on
an item that you hope will never loosen. In order to turn the bolt inward, the spanner needs to be rotated
clockwise. Two options are presented below.

Bolt Bolt

Option 1: 20 cm spanner Option 2: 10 cm spanner

Which spanner do you think would be more effective if both were applied
with the same force? The turning force depends on how far the force is
applied from the point of application, and so the first option of the longer
spanner would be the best choice.
This principle is observed in many other applications such as:
1. The location of the handle on a hinged door. If the handle was placed too
close to the hinges, a person would find it difficult to open the door.
In most instances the door handle is on the opposite side to the hinges
thereby increasing the distance from the point of application.
2. When a young child rides a bicycle with ‘coaster brakes’ they often try to
stop by applying a force at the peddle. The length of the crank arm, along
with the applied force, determines the magnitude of turning force applied
at the crank.

This turning force is referred to as torque, and is defined by:

Torque = |~r × ~
F|

where ~
F is the force and ~r is the vector from the point of application of the force to the point, or axis, about
which the turning is considered.

90 Jacaranda Maths Quest 12 Specialist Mathematics Units 3 & 4 for Queensland


“c02VectorsInThreeDimensions_print” — 2019/8/7 — 6:51 — page 91 — #70

As this equation suggests, the size of the torque depends not only on the force and the distance the force is
from the point of application, but also on the angle at which the force is applied. The torque is a maximum
when the angle between ~ F and ~r is 90°. If ~
F is measured in Newtons (N) and ~r is measured in metres (m)
then torque is measured in Newton metres (Nm).

WORKED EXAMPLE 33
The diagrams depict two different spanners being acted on by a force, F
~ n . For each scenario,
calculate the torque about the point O (~r is in metres).
a. y

r = 5î
~1

x
O
F1 = 30 ĵ N
~

b. y

r = 3î + 2ĵ
~2
70°

O x

F2 = 50 N
~

THINK WRITE

a. 1. Recall the torque formula. a.
~r = OP = 5î, ~ F = −30ĵ
Substitute in the vectors, Torque = |~r × ~ F|
~r and ~
F. = | (5î) × (−30ĵ) |
|î ĵ k̂ |
| 0 ||
2. Calculate the vector ~r × ~F = |5 0
product using the | 0 −30 0 |
determinant method. | 0 0| |5 0| |5 0|
= î| | − ĵ| | + k̂ | |
| −30 0 | |0 0| | 0 −30 |
= −150k̂
3. The vector product is in |~r × ~
F| = | − 150|
component form. = 150 Nm
Determine the magnitude The torque is 150 Nm.
of ~r × ~
F.

CHAPTER 2 Vectors in three dimensions 91


“c02VectorsInThreeDimensions_print” — 2019/8/7 — 6:51 — page 92 — #71


b. 1. Force needs to be b.
~r = OP = 3î + 2ĵ
expressed as a vector in F = 50 cos (70°) î − 50 sin (70°) ĵ
~
component form. = 17.10î − 46.98ĵ
F = |~
~ F| (cos 𝜃î + sin 𝜃ĵ)
2. State the torque formula. Torque = |~r × ~
F|
Substitute in the vectors, = | (3î + 2ĵ) × (17.10î − 46.98ĵ) |
~r and ~
F.
| î ĵ k̂ |
3. Calculate the vector F| = || 3
|~r × ~ 2 0 ||
product using the | 17.10 −46.98 0 |
determinant method. | 2 0| | 3 0| | 3 2|
= î| | − ĵ| | + k̂ | |
| −46.98 0 | | 17.10 0 | | 17.10 −46.98 |
= −175.14k̂
4. The vector product is in |~r × ~
F| = | − 175.14|
component form. = 175.14 Nm
Determine the magnitude The torque is 175.14 Nm.
of ~r × ~
F.

2.7.3 Applications of vector functions of time


Vector equations of lines, vector functions and parametric equations are convenient methods to describe the
position of a particle in two or three dimensions according to a given parameter, t. As shown in many of the
previous sections of work, time is a common parameter. The following examples explore various
applications of vector functions of time in both abstract and real life instances.

WORKED EXAMPLE 34
The paths of two model sail boats are defined
by the following vector functions of time, where
time is measured in minutes and distance in metres.

Boat A: ~r A (t) = (t + 13) î + 6tĵ


Boat B: ~r B (t) = (3t − 5) î + (5 + t) ĵ

a. Calculate the distance between the two boats


at t = 8 minutes.
b. Calculate if, at any given time t (seconds), they
are in the same location. That is, determine if there
is any possibility of a collision.

THINK WRITE
a. 1. Evaluate the position of boat A by letting a.
~r A (8) = (8 + 13) î + 6 × 8ĵ
t = 8, expressed as ~rA (8). = 21î + 48ĵ
2. Evaluate the position of boat B by letting r
~B (8) = (3 × 8 − 5) î + (5 + 8) ĵ
t = 8, expressed as ~rA (8). = 19î + 13ĵ

92 Jacaranda Maths Quest 12 Specialist Mathematics Units 3 & 4 for Queensland


“c02VectorsInThreeDimensions_print” — 2019/8/7 — 6:51 — page 93 — #72

The vector joining the two boats at


3. |~rB (8) − ~r A (8) | = |(19î + 13ĵ) − (21î + 48ĵ)|
t = 8 min, ~rB (8) − ~rA (8), defines the = |−2î − 35ĵ|

directed line segment between the two
= (−2)2 + (−35)2
boats. The magnitude of the resultant √
vector is then calculated. = 1229
This represents the straight line distance The boats are approximately 35.06 metres
between boat A and B. apart.
b. 1. The boats will collide if they reach the b.
~r A = ~r B
same point at the same time. (t + 13) î + 6t ĵ = (3t − 5) î + (5 + t) ĵ
Equate the two vector functions.
2. Equate the î and ĵ components and solve î component: ĵ component:
for t in both cases. t + 13 = 3t − 5 6t = 5 + t
18 = 2t 5t = 5
2t = 18 t=1
t=9
3. State your observations. There is no common time for when the two
functions are in the same location.
4. State the answer. The boats do not collide.

WORKED EXAMPLE 35
The location of two particles can be defined by the following vector functions of time.
( 2
) ( 2) ( 3
)
~r A (t) = (144 − t î )+ 2t ĵ + 0.25 (t(− 2) k̂ )
2 2
~r B (t) = t − 5t − 6 î + (240 − 4t) ĵ + (t − 4) + 92 k̂

Calculate if, at any given time t, t ≥0 seconds, they are in the same location.

THINK WRITE
1. The particles will be in the ~r A = ~r B
same location if they reach (144 − t2 )î + 2t2 ĵ + (0.25(t − 2)3 )k̂ = (t2 − 5t − 6)î
the same point at the same
time. +(240 − 4t)ĵ + ((t − 4)2 + 92)k̂
Equate the two vector
functions.
2. Equate the î and ĵ î component: ĵ component:
components and collect 2 2
144 − t = t − 5t − 6 2t2 = 240 − 4t
like terms.
0 = 2t2 − 5t − 150 2t2 + 4t − 240 = 0
t2 + 2t − 120 = 0
3. Factorise both quadratic 0 = (2t + 15)(t − 10) (t − 10)(t + 12) = 0
equations and apply the (2t + 15) = 0 or (t − 10) = 0 (t − 10) = 0 or (t + 12) = 0
Null factor Law to solve t = −7.5 or t = 10 t = 10 or t = −12
for t.

CHAPTER 2 Vectors in three dimensions 93


“c02VectorsInThreeDimensions_print” — 2019/8/7 — 6:51 — page 94 — #73

4. State your observations. As time cannot be negative, we can reject the solutions of
t = −7.5 and t = −12.
There is a common time in which the two particles are in the
same location. This occurs when t = 10 seconds.
( 2
) 2
( 3
)
5. Determine the position ~r A (10) = 144 − (10) î + 2(10) ĵ + 0.25(10 − 2) k̂
vector of both particles at = 44î + 200ĵ + 128k̂
( 2 ) ( )
t = 10. 2
~r B (10) = 10 − 5(10) − 6 î + (240 − 4(10))ĵ + (10 − 4) + 92 k̂
= 44î + 200ĵ + 128k̂
6. Summarise findings. The two particles will be in the same location when t = 10
seconds. The position vector at this location is
44î + 200ĵ + 128k̂.

WORKED EXAMPLE 36
The parametric equation, x = 4 − t, y = 1 + 3t, z = 4 + 2t, defines the path of particle.
Determine the point of intersection between the particle and the plane, 2x + 4y − 2z = 1.

THINK WRITE
1. Define the three parametric equations as [1] to [3]. x= 4−t [1]
y = 1 + 3t [2]
z = 4 + 2t [3]
2. The point of intersection must satisfy both the 2 (4 − t) + 4 (1 + 3t) − 2 (4 + 2t) = 1
equation of the plane and line. Substitute [1], [2] 8 − 2t + 4 + 12t − 8 − 4t = 1
and [3] into the equation of the plane. Continue by 6t + 4 = 1
solving for t. 1−4
t=
6
t = −0.5
3. Evaluate the parametric equation, for t = −0.5 to x = 4 − (−0.5)
calculate the point of intersection. = 4.5
y = 1 + 3 (−0.5)
= −0.5
z = 4 + 2 (−0.5)
=3
4. State the solution as the point of intersection. The particle intersects the plane at
(4.5, −0.5, 3)

Units 3 & 4 Area 2 Sequence 1 Concept 6


Applications of vectors Summary screen and practice questions

94 Jacaranda Maths Quest 12 Specialist Mathematics Units 3 & 4 for Queensland


“c02VectorsInThreeDimensions_print” — 2019/8/7 — 6:51 — page 95 — #74

Exercise 2.7 Applications of vectors


Technology active
1. WE30&31 Calculate the equation of the plane perpendicular to n = n1 î + n2 ĵ + n3 k̂, containing the point
~
P0 = (x0 , y0 , z0 ).
a. P0 = (3, 6, −2) and n = 5î − 2ĵ + k̂ b. P0 = (−2, 1, 4) and n = −3î + 5ĵ + 4k̂
~ ~
c. P0 = (−1, 0, 3) and n = −7î + 4ĵ − 6k̂ d. P0 = (4, −1, −4) and n = −4î − ĵ + 3k̂
~ ~
2. Determine a unit vector, ~ n̂ , to each of the following planes.
a. 4x − 2y − z = 12 b. −x + 2y − 12 = 2z
c. 4 = −6x + 7y − 3z
3. WE32 a. Calculate the Cartesian equation of the plane determined by the points below.

b. Determine the axis intercepts for each plane.


i. P (1, 4, 2), Q (2, 3, 5) and R(1, 9, 8) ii. P (−2, 5, 3), Q (3, −1, 2) and R(−7, 1, 0)
iii. P (−1, 4, 8), Q (1, 8, −3) and R(12, 9, −5) iv. P (−6, 7, 1), Q (−9, 2, 4) and R(3, −3, 8)
4. For each of the following, determine if the point P (x, y, z), lies on the given plane.
a. P (2, 9, −1), plane: 4x − 2y + z = −11
b. P (−1, −6, 4), plane: 3x − z + 1 = 10
c. P (1, 4, −7), plane: −x + 3y + 5z = −24
5. Two planes are parallel if both normal vectors are parallel. Similarly, if two planes are perpendicular,
then so too are their normal vectors.
a. Examine the two planes and determine if they are parallel. P1 : 4x − 2y + 4z = 15 and
P2 : 2x − y + 2z = −23
b. Determine the equation of the plane that is parallel to 11x − 2y + 6z = 12 and contains the point
P (1, −8, 2).
c. Calculate the value of c if the two planes, P1 : 8x − 6y − 2z = 1 and P2 : 2x − y + cz = −6, are
perpendicular.
6. The angle between two planes is calculated using the normal vectors ~n1 and ~n2 . The angle can be
|n1 · n2 |
determined by the formula, cos 𝜃 = ~ ~ . Calculate the angle between the two planes,
|~n1 | × |~n2 |
4x − y + 2z = 0 and 2x + 3y − z = −1 to three decimal places.
7. Explain why the equation of a plane cannot be formed by the points P (−2, 6, −18), Q (−12, 8, −68)
and R (13, 3, 57).
8. a. Calculate the equation of the plane that is perpendicular to the plane 8x + 2y + z = 0, passing
through the points P (2, 3, 4) and Q (5, 7, 8).
b. Justify that the intersection of the two planes form a right-angle.
9. Triangular shaped shade sails are often used in parks
and schools. The vertices of a shade sail can be
located by the points L (3, −5, 3), M (−4, −1, 4) and
N (1, 2, 5), where all units are in metres.
a. Develop an equation to model the plane created by the
erected triangular shade cloth. State the assumptions
considered true when developing models and solutions.
b. Calculate the area of the shade cloth to two decimal
places.
10. a. A plane is perpendicular to the vector n and contains
~ |n · ⃗PQ |
the point P. Prove that the shortest distance, D, from the plane to a point Q is given by D = ~ .
|~n|

CHAPTER 2 Vectors in three dimensions 95


“c02VectorsInThreeDimensions_print” — 2019/8/7 — 6:51 — page 96 — #75

|n · ⃗PQ |
b. Apply the formula, D = ~ , to calculate the shortest distance from Q (6, 5, 4) to the plane,
|~n|
x + 5y − 2z = 8. (Hint, P (x, y, z) is any point on the plane)
c. If a plane is in the form, ax + by + cz = d, the shortest distance to a point Q = (x1 , y1 , z1 ) can be
|ax1 + by1 + cz1 − d|
calculated by the equation, D = √ . Use this alternative approach to verify the
a2 + b2 + c2
solution obtained in Question 10b.
11. A line is formed when two non-parallel planes intersect. A vector in the direction of the line of
intersection can be calculated by the vector product, ~n1 × ~n2 . This implies that the vector product of the
two normals are parallel to the line of intersection. Calculate the parametric equation for the line of
intersection of the planes, 8x − 9y − 2z = 4 and −6x + 2y − z = 13.
12. WE33 A force, F, acts on an object at a point, P. In each of the following cases, calculate the torque
~
about O. The coordinate location of points is measured in metres.
a. P (10, 0, 0) and F = −20k̂N
~ ( )
b. P (6, 1, −2) and F = 4î − 7ĵ + k̂ N
~ ( )
c. P (2, −5, 6) and F = −5î + 2ĵ − 3k̂ N
~
13. To rotate the level arm shown on the right, the turning P (5, 3, 0)
force must be greater than 400 Nm. Calculate ( the torque
) 𝜃 = 22°
about the point O, given the force, ~ F = xî + yĵ + zk̂ ,
acting on the object at P.
F = 150 N
14. A metal bracket is fixed to the wall at a rotatable point O. ~
The force, ~F, is applied at P. The magnitude of the torque
at point O is 1586.7 Nm.
O (0, 0, 0)
Calculate the vector location of point P.

1m P
θ = 50°

F = 400 N
~
O 7m

5m

15. A force of 400 N is applied on the cube at point A, as shown. Calculate the torque of the force about B to
two decimal places.

~F

4m

4m
B

4m A

96 Jacaranda Maths Quest 12 Specialist Mathematics Units 3 & 4 for Queensland


“c02VectorsInThreeDimensions_print” — 2019/8/7 — 6:51 — page 97 — #76

16. WE34 The paths of two model sail boats are defined by the following vector functions of time, where

time is measured in minutes and distance in metres.

Boat A: ~rA (t) = (9t − 7)î + 11t ĵ


Boat B: ~rA (t) = (2t + 14)î + (9 + t)ĵ

a.Calculate the distance between the two boats at t = 5 minutes.


b.Calculate if, at any given time t (minutes), they are in the same location. That is, determine if there is
any possibility of a collision.
17. WE35 The flight path, measured in metres, of two
helicopters is given by the vector functions of time
(minutes) below.
( 2
)
r
~ A (t) = 169 + 0.5t î + (10t + 120) ĵ +
( )
0.45t3 + 2 k̂
( 2
) ( 2 )
r
~ B (t) = (4t + 179) î + 240 − 0.2t ĵ + 2t + 20 k̂

Determine if the helicopters will ever cross paths.


State any assumptions used in developing models
and solutions.
18. Two particles are moving in a circular motion in a three-dimensional space. The position of the first
particle is defined by

xa = 6 sin t , ya = 4 cos t , za = 5 0 ≤ t ≤ 10𝜋

The position of the second particle is defined by

xb = 2 cos t − 6 , yb = 2 + 2 sin t , zb = 5 0 ≤ t ≤ 10𝜋

The position of both particles are functions of time (t) given in seconds. Calculate the time and position
of when the two particles first intersect.
19. WE36 The parametric equation, x = 2 + 4t, y = −4 − 2t, z = 3 + 3t, defines the path of particle.
Determine the point of intersection between
( the particle
) and ( the plane, )−4x + 3y + 7z = 21.
20. The vector equation of the line, r = −3î + 5ĵ − 2k̂ + k 2î + ĵ − 6k̂ , defines the path of particle.
~
Determine the point of intersection between the particle and the plane, 7x + 4y + 8z = 1.

2.8 Review: exam practice


A summary of this chapter is available in the Resources section of your eBookPLUS at
www.jacplus.com.au.
Simple familiar
1. MC Consider the points P (−2, 5, 1), Q (5, 8, 4) and R (9, −6, 12). The resultant vector of the vector
sum, 2 ⃗
OP + ⃗QP − 4 ⃗ QR , is:
A. −43î + 56ĵ − 30k̂ B. −27î + 63ĵ − 33k̂
C. −10î + 57ĵ − 27k̂ D. −42î + 61ĵ − 35k̂
2. MC Determine the angle between the given vectors, a = 2î − 5ĵ + 8k̂ and b = 9î + 7ĵ − 8k̂.
~ ~
A. 𝜃 = 129.24° B. 𝜃 = 127.81°
C. 𝜃 = 127.09° D. 𝜃 = 128.56°

CHAPTER 2 Vectors in three dimensions 97


“c02VectorsInThreeDimensions_print” — 2019/8/7 — 6:51 — page 98 — #77

3. MC When expressed in component form, the vector u = [15, 325°, 36°] is defined by which of the
~
following?
A. u = 9.94î − 6.96ĵ + 8.82k̂ B. u = 10.25î − 7.14ĵ + 9.72k̂
~ ~
C. u = 11.36î − 6.07ĵ − 8.01k̂ D. u = 11.87î − 5.99ĵ + 7.67k̂
~ ~
t2
4. MC Determine the equivalent Cartesian equation for x = 2t − 7, y = +t
7
1 2 21
A. y = x + 2x −
4 2
1 2 15
B. y = x +x+
16 2
1 2 21
C. y = x + 2x +
26 8
1 2 21
D. y = x +x+
28 4
5. MC A sphere contains the points, A (7, 13, 8) and B (−1, 3, 2), that are the endpoints of its diameter.
Calculate the Cartesian equation of the sphere.
2 2 2 2 2 2
A. 45 = (x + 3) + (y − 8) + (z − 5) B. 50 = (x − 3) + (y − 8) + (z − 5)
2 2 2 2 2 2
C. 52 = (x − 3) + (y + 8) + (z − 6) D. 49 = (x − 2) + (y − 7) + (z − 6)
6. a. Calculate the vector equation for the line passing through the given points, A (5, −2, 13) and
B (6, 4, 7).
b. Determine the value for y × z if the head of vector, c = 12î + yĵ + zk̂, touches the line at P (x, y, z).
~
7. MC Calculate a × b for the given vectors, a = [19, 7°] and b = [24, 145°].
~ ~ ~ ~
A. a × b = 305.12 n̂
~ ~ ~
B. a × b = 299.82 n̂
~ ~ ~
C. a × b = −311.89 n̂
~ ~ ~
D. a × b = 309.74 n̂
~ ~ ~
8. Calculate a vector perpendicular to both ~a = 5î + 6ĵ − 4k̂ and ~b = 4î − 9ĵ + 7k̂.
9. Determine the volume of the parallelepiped containing the adjacent vectors,
~ = 4î + 4ĵ + 2k̂, ~b = 8î + 6ĵ − 3k̂ and ~c = 12î − 5ĵ − 7k̂.
a
10. Determine the equation of the plane containing the point (−2, 9, 1) if the vector ~n = 3î + 5ĵ + 8k̂ is
normal to it.
11. The path (measured in metres) of two objects in flight is given by the following vector functions of time
(minutes):
(2 ) ( 2
)
~r a (t) = t + 4 î + (4t + 9) ĵ + 0.5t + 1 k̂, 0 ≤ t ≤ 20
( 2
)
~r b (t) = (8t + 184) î + 0.25t ĵ + (3t + 109) k̂, 0 ≤ t ≤ 20

Determine if the particles will cross paths at the same time, stating the time it occurs and the location
at that point in time.
12. The parametric equation, x = 2 − t, y = 2 + t, z = 5 + 2t, defines the path of particle.
Determine the point of intersection between the particle and the plane, x + 2y − 3z = −14.

Complex familiar
13. The vector product of two vectors a = −2î + 11ĵ − k̂ and b = −3î + by ĵ + bz k̂ is
~ ~
a ×
~ ~ b = 12î + 5ĵ + 31k̂. Calculate the values of by and b z .
14. Calculate the equation of the plane determined by the following points: P (3, 8, 9), Q (4, −8, −5) and
R(−5, 6, 2).

98 Jacaranda Maths Quest 12 Specialist Mathematics Units 3 & 4 for Queensland


“c02VectorsInThreeDimensions_print” — 2019/8/7 — 6:51 — page 99 — #78

15. From base camp, a group of explorers can observe a


famous ruin that can be located by the position vector,
~v = 150î + 160ĵ + 650k̂, where all units are in metres.
Convert the position vector to polar form.
Interpret the both the magnitude and altitude angle to
assess if the path from camp to the ruin would
be reasonable.
16. Given a = 3î + 2ĵ + 9k̂, b = 8î + 3k̂ and
~ ~
~c = 6î + 2ĵ + 6k̂, prove (that the
) vector
( product
) is not
associative. That is, ~a × ~b × c ≠ ~a × ~b × ~c.
Complex unfamiliar
17. A golfer is trying to hit the golf ball as far as possible from ground level. The path of the golf ball,
according to time t (seconds), can be modelled by the parametric equations,

( ) ( ) 1
x = v0 cos 𝜃 t, y = v0 sin 𝜃 t − gt2 (where g = 9.8)
2

When the golfer strikes the ball, 𝜃 = 30° and v0 = 66 ms−1 .


Calculate the maximum height the ball reaches and when it strikes the ground. State any assumptions
considered to be true when developing models and/or solutions. Justify the reasonableness of the results
by considering the location of the golf ball when it returns to ground level.

18. Consider the two vectors, ~a and ~b, in the diagram below.

a
~ b
~

θ1
θ2
x

( )
Verify the identity sin 𝜃a − 𝜃b = sin 𝜃a cos 𝜃b − cos 𝜃a sin 𝜃b . The proof should be constructed by
calculating the vector product, ~a × ~b.
19. Calculate the equation of the plane containing the point P (−3, 2, 5), and containing the line of
intersection of the planes x − 2y + 4z = 3 and −3x + 5y − 4z = −5. Technology can be used to
determine a particular point on the line formed by the intersection of the planes.

CHAPTER 2 Vectors in three dimensions 99


“c02VectorsInThreeDimensions_print” — 2019/8/7 — 6:51 — page 100 — #79

20. In the triangle ABC, D divides AC in the ratio 2:1, E divides CB in the ratio 2:1 and F divides BA in the
1
ratio 2:1. Prove that the area of the triangle GHI is times the area of triangle ABC.
7
C

D
H

G E
A
I
F

Units 3 & 4 Sit exam

100 Jacaranda Maths Quest 12 Specialist Mathematics Units 3 & 4 for Queensland
“c02VectorsInThreeDimensions_print” — 2019/8/7 — 6:51 — page 101 — #80

Answers 20. The resultant displacement of the boat from its starting
position is 939.692 km, in the direction, 160°T.
21. 𝜆 = −2; 𝜆a = −12î − 4ĵ + 2k̂
2 Vectors in three dimensions ~
22. The helicopter has travelled 269.14 metres when it reaches
Exercise 2.2 Introduction to vectors in three the closest point. The distance to the mountain climber at
dimensions that point is 8.13 metres.
1. a. A (5, −4) b. (−2, 6)
( ) Exercise 2.3 Geometric proofs using vectors
5
c. C , 11, −12 d. D (−4, −1, 8) 1, 2. Sample responses can be found in the worked solutions in
2
the online resources.
2. a. 5î + 3ĵ + 2k̂ b. 2î − 6k̂ 1√
2 2 22 3. 870 units squared
c. − î + ĵ − k̂ d. −11î + ĵ − 4k̂ 2
5 3 17 4–16. Sample responses can be found in the worked solutions
3. a. Vector b. Scalar in the online resources.
c. Vector d. Scalar
e. Scalar Exercise 2.4 Cartesian and parametric
4. a. −4î + 2ĵ − 7k̂ b. −6î + 10ĵ − 2k̂
equations
c. î + 3ĵ + 6k̂ d. 22î − 18ĵ + 30k̂
1. a. A (2, 2, 0) b. B (2, −4, −2)
5. a. ⃗CE = r + t ⃗I = r + t + s
b. C
~ ~ ~ ~ ~ c. C (2, −4, 0) d. D (0, 4, 0)
c. ⃗
DJ = −r + s + t e. E (−3, 4, 0)
~ ~ ~ √
6. The aeroplane is 1081.67 km at a bearing 146.31°T from its 2. a. 274
starting location. √
b. 146
7. a. 8î + 4ĵ + k̂ b. 9 √
1( ) c. 233
c. 8î + 4ĵ + k̂ √
9 d. 206

a. 6î + 2ĵ − 8k̂ b. 2 26 3. a. (7, 7, −1) b. (−3, 4, −7)
8. ( )
1 ( ) 7 ( ) 11 −9
c. √ 3î + ĵ − 4k̂ d. √ 3î + ĵ − 4k̂ c. (4, −6, 4) d. , −2,
2 2
26 26
2
( )2
9. a. 5î + ĵ + 8k̂ b. î + 2ĵ + k̂
4. a. 36 = (x − 2) + y − 8 + (z + 3)2
2
( )2
c. −17î + 2ĵ − 29k̂ b. 9 = (x + 3) + y − 1 + (z + 9)2
10. 5î + 6ĵ − k̂ ( )2
c. 49 = (x − 1)2 + y + (z − 6)2
11. a. −2î + 5ĵ ( )2
b. 2î + 3ĵ + 4k̂ d. 11 = (x + 9)2 + y + 2 + (z − 7)2
c. −5î + 48ĵ − 16k̂
( )2
5. a. 50 = (x − 3)2 + y − 8 + (z − 5)2
d. 2î + 11ĵ + 8k̂, ∴ M (2, 11, 8) ( )2
b. 56 = (x + 1)2 + y + 1 + (z − 5)2
PQ = ⃗
e. 3 ⃗ QS Therefore, the vectors are parallel. ( )2
12. a. 15.73 c. 254 = (x + 3)2 + y + 2 + (z + 10)2
b. −5.87 ( )2
d. 45 = (x − 5)2 + y + 6 + (z − 3)2
13. −45 6. a.
2 2 2
x + y + z − 10x − 14y − 2z − 69 = 0
14. y = −18 b. x2 + y2 + z2 + 2x − 6y + 4z − 67 = 0
15. −76 = ( −76) c. x2 + y2 + z2 + 6x − 2y − 5 = 0
∴ c· a−b =c·a−c·b x2 + y2 + z2 − 2x − 8y + 6z − 30 = 0
~ ~ ~ ~ ~ ~ ~ d.
16. 𝜃 = 75.29° 7. a. C (5, −2, −6), r = 9
−94 940 846 376 b. C (−1, −4, 8), r = 9
17. a. √ b. − î+ ĵ− k̂ c. C (−10, 6, −12), r = 10
197 197 197 197
d. C (−1, −2, −3), r = 5
45 58 18 2 11
c. − î− k̂ĵ− 8. a. y= x+
197 197 197 3 3
18. a. 2.47î + 4.45ĵ + 3.18k̂ b. −7.81î + 3.98ĵ − 17.98k̂ b. y = −x2 − 9x − 20
c. −1.15î + 1.99 ĵ − 1.93k̂ d. 9.94î − 6.96ĵ + 8.82k̂ c. y = x2 + 2x + 8
19. i. a. 𝜑 = −29.12° (√ )2
[√ ] 3
x 1 2
b. 38, 68.20°, −29.12°
d. y= or y = x 3
ii. a. 𝜑 = 21.80° 4 4
[√ ] 4x − 17
b. 29, 306.87°, 21.80° e. y=
x−4
iii. a. 𝜑 = 41.87°
[√ ] 4 − x2
b. 110, 50.19°, 41.87° f. y= = −0.25x2 + 1
4
iv. a. 𝜑 = −57.37°
[√ ]
b. = 141, 128.66°, −57.37°

CHAPTER 2 Vectors in three dimensions 101


“c02VectorsInThreeDimensions_print” — 2019/8/7 — 6:51 — page 102 — #81

2
9. a. x + y = 1
2 y
2 2
b. x + y = 1 24
22
x2 y2
c. +=1 20
16 49 18
2 2
d. x −y =1 16
x 14
10. a. y=
15 12
x2 x 10
b. y= − 8
16 4 (0,7)
x3 6
c. y= +2 4
27 2
2 2
d. x +y =4
x
11. a. î + 3.5ĵ –2 –1 0 1 2 3 4
b.
–2
t x (t) y (t)
~r (t) c. 2î + 71ĵ
14. r (t) = (3 − 5t)î + (2 + 3t)ĵ + (t + 1)k̂, for t ∈ R
0 0 0 0î + 0ĵ ~
1 1 3.5 î + 3.5ĵ
t x (t) y (t) z (t)
~r (t)
2 2 6 2î + 6ĵ
1 −2 5 2 −2î + 5ĵ + 2k̂
3 3 7.5 3î + 7.5ĵ
3 −12 11 4 −12î + 11ĵ + 4k̂
4 4 8 4î + 8ĵ
5 −22 17 6 −22î + 17ĵ + 6k̂
12. a. 6î + ĵ
b.
t x (t) y (t) b. The path of the particle could be described as a vector
~r (t) equation of a line.
0 0 0 0î + 16ĵ 15. a. Sketch the plot using 3D software.
z
1 2 9 2î + 9ĵ

2 4 4 4î + 4ĵ 10 r
~a
3 6 1 6î + 1ĵ

4 8 0 8î

5 10 1 10î + ĵ
O
2
x y
c. y = − 4x + 16
4
0 ≤ x ≤ 10, 0 ≤ y ≤ 16 –2 r
~b
d. y
(0,16)
16 x
14
12
10
8 b.
6
~r a (t) is a circle with radius, r = 5 shifted 10 units up
4 ~r b (t) is a circle with radius, r = 6 the circle has been
2
(10,1) shifted 2 units downward
16. a.
–2 0 2 4 6 8 10
x ~r (0) = 0î + 0ĵ + 0k̂
–2 ~r (12) = 36î + 34.56ĵ + 5.6k̂
3 b. 50.22 units
13. a. y = 8x + 7
17. a. 850î + 950î + 350k̂
b. 0 ≤ x ≤ +∞ and 7 ≤ y ≤ +∞
b. The largest value for b = 32.71
c. 633.75 m
d. No, it never ascends in flight path, it is always
descending.

102 Jacaranda Maths Quest 12 Specialist Mathematics Units 3 & 4 for Queensland
“c02VectorsInThreeDimensions_print” — 2019/8/7 — 6:51 — page 103 — #82

18. a. x = cos(4t), y = sin(6t) for t ≥ 0 y


y 20
2
15
1
10

π 0 π x
–π –— — π 5
2 2
–1

0 x
–2 –20 –15 –10 –5 5 10 15 20
–5
b. x = sin 5t
y = cos t for t ≥ 0 –10
y

–15
1

–20

b. r (t) = 4 (t − sin(t)) î + 4 (1 − cos(t)) ĵ,


~
for −2𝜋 ≤ t ≤ 4𝜋
π
–— π
–—
0 π
— π x

2 4 4 2 *See image at foot of the page
2
c. r (t) = 2 cot(t) î + 2 sin (t) ĵ,
~
for −𝜋 ≤ t ≤ 𝜋
–1 y
10

c. x = − cos 2t, 5
y = sin 3t for t ≥ 0
y
0 x
–4π –2π 2π 4π
1
–5

d. r (t) = 3 cos(t) î + 2 sin(2t) ĵ,


~
for −𝜋 ≤ t ≤ 𝜋
y
π
–— π
–—
0 π
— π x
— 2
2 4 4 2

1
–1

π 0 π x
–π –— — π
19. a. r (t) = t cos(t) î + t sin(t) ĵ, 2 2
~ –1
for 0 ≤ t ≤ 6𝜋
–2

20. Sample responses can be found in the worked solutions in


the online resources.

*19. b y
10

0 x
–10π –8π –6π –4π –2π 2π 4π 6π 8π 10π 12π 14π 16π 18π
–5

–10

CHAPTER 2 Vectors in three dimensions 103


“c02VectorsInThreeDimensions_print” — 2019/8/7 — 6:51 — page 104 — #83

Exercise 2.5 The vector equation of a d. 0


straight line 2. a. 91.93n̂
( ) ( ) ~
b. 334.72n̂
1. a. r = 2î + 1ĵ + k 4î + 6ĵ
~ ( ) ~
( ) c. 13.57n̂
b. r = 6ĵ + k 9î − 18ĵ
~
~ ( d. 22.50n̂
) ( ) ~
c. r = −4î + 6ĵ + k 3î + 6ĵ
~ ( 3. a. 12.04 units2
) ( ) b. 7.43 units2
d. r = −2î − 4ĵ + k 7î + 7ĵ
~ ( ) ( ) 4. a. 5.2 units2
e. r = 2î + 8ĵ + k 5î − 9ĵ 28.39 units2
~ ( ) ( )
b.
f. r = −7î − 5ĵ + k 8î + 20ĵ 5.
~ ( ) ( )
× î ĵ k̂
2. a. r = −2î + 8ĵ + k 7î − 11ĵ
~ î k̂ −ĵ
b. 19î − 25ĵ ~0
3. a. P (10, −8) ĵ −k̂ î
b. P (1, 3)
~0
( ) ( ) k̂ ĵ −î
4. a. r =
~ −2î + 2ĵ + k̂ + k 2î + 4ĵ + 6k̂ ~0
( ) ( ) 6. a. 43î + 11ĵ + 17k̂
b. r = 4î − 1ĵ + 11k̂ + k −17î + 10ĵ − 23k̂
~ ( ) ( ) b. −10î − 43ĵ + 31k̂
c. r = 5î + 4ĵ + 11k̂ + k −6î − 3ĵ − 3k̂ c. 60î − 79ĵ + 111k̂
~ ( ) ( )
d. r = 6î − 1ĵ + 2k̂ + k −î + 4ĵ − 11k̂
31
~ ( d. −4î − ĵ − k̂
) ( ) 4
e. r = −3î − 2ĵ + k̂ + k 8î + 9ĵ − 14k̂
~ ( ) ( )
7. a. a × b = 7î + 10ĵ + 29k̂
~ ~
f. r = î + 5ĵ + 8k̂ + k 6î − 5ĵ − 9k̂ b. b × a = 9î − ĵ − 32k̂
~ ( ) ( ) ~ ~
5. a. r = −4î + 7ĵ + 6k̂ + k 10î − 4ĵ − 4k̂ c. Sample responses can be found in the worked solutions
~ in the online resources.
b. C (96, −33, −34) does lie on the line.
8. A unit vector normal to vectors a and b is not possible. The
c. z = 18 ~ ~
two vectors do not define a plane as they are scalar
6. a. 27î − 24ĵ − 15k̂
multiples of each another.
b. 0.5î − 1ĵ − 14k̂
9. a. −9î − 46ĵ − 59k̂
7. a. P (12, 29, 0) does lie on the line.
b. a × b ≠ b × a. Not commutative.
b. P (−3, −28, 2) does not exist on the line. ~ ~ ~ ~
c. P (4, 10, 13) does exist on the line 10. a. 19î − 38ĵ + 19k̂
( ) ( )
8. r = 6î + 5ĵ + k̂ + k 6î − 2ĵ + 3k̂ b. 69î + 32ĵ + 12k̂
~ ( ) ( ) c. 10î − 25ĵ + 20k̂
9. r = î + 2ĵ + 4k̂ + k k̂ d. −59î + 55ĵ + 78k̂
~
10. x = −2 and z = 10 11. a. 246 units
3
b. 172 units
3

11. a. x = −1 + 2k, y = 3 + 5k, z = 7 − 3k c.√164 units


3
d. 276 units
3

b. x = −7 + 19k, y = 8 + 11k, z = 3 + k 12. 6 59 units


2

c. x = 5 + 2k, y = 1 + 2k , z = −3 − 6k 13. Sample responses can be be found in the Worked solutions


12. a. x − 1 = y − 2 = z − 3 in the online resources.
x+2 4−y 8−z 14. The vectors are coplanar as the scalar triple product equals
b. = =
6 9 2 zero.
x − 11 y−3 z − 12 15. bz = 1 and by = −4
c. = =
12( 21 ) 16. a. 0
(13 )
13. a. r = 2î + 5ĵ − 8k̂ + k 3î + 2ĵ + 5k̂ b. 1
~ ( ) ( ) c. −1
b. r = −7î − ĵ + 2k̂ + k −2î + 6ĵ + 3k̂
( ~ ) d. −1
34 41 17. 68.199°
14. a. , ,0 3
5 5 18. 2.61 units
( ) 19. 30.61 units
2
44 15
b. − , 0,
6 6
√ Exercise 2.7 Applications of vectors
147
15. D = √ ≈ 4.95 units 1. a. 5x − 2y + z = 1
6 b. −3x + 5y + 4z = 27
16. The aircraft do not cross paths. c. −7x + 4y − 6z = −11
17. 𝜃 = 39.92° d. −4x − y + 3z = −27
1 ( )
Exercise 2.6 The vector product 2. a. √ 4î − 2ĵ − k̂
21
1. a. 48.86 n̂ out of the page
~ 1( )
b. 14.854 n̂ into the page b. −î + 2ĵ − 2k̂
~ 3
c. 52.5n̂ into the page
~

104 Jacaranda Maths Quest 12 Specialist Mathematics Units 3 & 4 for Queensland
“c02VectorsInThreeDimensions_print” — 2019/8/7 — 6:51 — page 105 — #84

1 ( )
c. √ b. rA (3) ≠ rB (3). No collision.
6î − 7ĵ + 3k̂ ~ ~
94 17. r (10) = 219î + 220ĵ + 452k̂
3. i. a. 21x + 6y − 5z = 35
~
3𝜋
5 35 18. t = seconds
b. x = , y = , z = −7 2
3 6 19. (−78, 36, −57)
ii. a. 14x + 20y − 50z = −78
20. (−4.2, 4.4, 1.6)
−39 −39 −39
b. x = ,y = ,z =
7 10 −25 2.8 Review: exam practice
iii. a. 3x − 117y − 42z = −807
269 269 1. B
b. x = −269, y = ,z = 2. C
39 14
iv. a. −5x + 48y + 75z = 441
3. A
147 147 4. D
b. x = −269 , y = ,z= 5. B ( ) ( )
16 25 6. a. r = 5î − 2ĵ + 13k̂ + k î + 6ĵ − 6k̂
4. a. P (2, 9, −1) exists on the plane. ~
b. y × z = −1160
b. P (−1, −6, −4) doesn’t exist on plane.
7. A
c. P (1, 4, −7) exists on the plane.
8. 6î − 51ĵ − 69k̂
5. a. n1 = 2n2 ∴ The planes are parallel
~ ~ 9. 372 units3
b. 11x − 2y + 6z = 39
10. 3x + 5y + 8z = 47
c. 11
11. r b (18) = ra (18) = 328î + 81ĵ + 163k̂,
6. 79.923° ~ ~
12. P(1, 3, 7)
7. Points are collinear. A plane cannot be formed.
13. bz = 1 and by = 1
8. a. 4x − 29y + 26z = 25
14. 84x + 119y − 130z = 34
b. P1 ⊥ P2
15. The straight-line distance is 686 metres. The altitude angle
9. a. x + 12y − 41z = −180
2 is 71.36°. This may be considered very steep and hence
b. 21.37m
unreasonable to climb.

10. a. |D| =
~n · PQ . Sample responses can be found in the 16. Sample responses can be found in the worked solutions in
|n| the online resources.
~
Worked Solutions in the online resources. 17. Horizontal distance is 384.94 metres. The maximum height
b. 2.74 units is 55.56 metres which occurred at t = 3.367 seconds. The
c. Sample responses can be found in the Worked Solutions result is unreasonable in terms of general driving distances.
in the online resources. It is assumed that no other forces, aside from gravity, act on
11 32 the ball during its flight, that the ground is level and no
11. x = − + 13k, y = 20k, z = − − 38k
10 5 spin/sideways movement occurs.
12. a. 200 Nm 18. Sample responses can be found in the worked solutions in
b. 49.81 Nm the online resources.
c. 32.03 Nm 19. 34x − 58y + 56z − 62 = 0
13. 698.19 Nm > 400 Nm, hence sufficient force. 1
14. ⃗
OP = 3.5î + 2ĵ 20. the area of ∆ABC. Sample responses can be found in the
7
15. 1306.39 Nm worked solutions in the online resources.
16. a. The boats are approximately 43.32 metres apart.

CHAPTER 2 Vectors in three dimensions 105

You might also like